数论-整除与同余

\chapter{数论}

\begin{verse}
古今之成大事业、大学问者,罔不经过三种之境界:“昨夜西风凋碧树。独上高楼,望尽天涯路。”此第一境界也。“衣带渐宽终不悔,为伊消得人憔悴。”此第二境界也。“众里寻他千百度,蓦然回首,那人却在,灯火阑珊处。”此第三境界也。——王国维《人间词话七则》
\end{verse}

\section{整除}

数论中我们只考虑整数问题,两个整数的加减乘法运算都是得到整数,为了让除法也能得到整数,我们引入整除定义.

\begin{definition}{}{dy1}
对于整数$a,b$且$b\neq 0$,若存在整数$q$使得$a=bq$,则称\textbf{$b$整除$a$}或\textbf{$a$可被$b$整除},记作$b\mid a$.此时,我们把$b$叫作$a$的\textbf{因数}或\textbf{约数},把$a$叫作$b$的\textbf{倍数}.
\end{definition}

\begin{theorem}{}{sl1}
\begin{enumerate}
\item (传递性)若$a\mid b,b\mid c$,则$a\mid c$.
\item 若$a\mid b$且$a\mid c$,则$a\mid b+c$.一般地有$a\mid xb+yc$,这里$x,y$均为整数.
\end{enumerate}

\end{theorem}

\begin{theorem}{带余数除法}{sl2}
若$a,b$均为整数且$b>0$,则存在两整数$q$和$r$,使得\[a=qb+r,\qquad 0\leq r<b\]成立,且$q$和$r$是唯一的.
\end{theorem}
此时称$q$为$a$被$b$除所得的\textbf{不完全商}, $r$叫作$a$被$b$除所得到的\textbf{余数}.

\begin{example}
\[20=4\times 5+0,\quad 17=3\times 5+2,\quad -13=-3\times 5+2.\]
\end{example}

 

\begin{example}
\begin{enumerate}
\item 证明:
\[(2+1)(2^2+1)(2^{2^2}+1)\cdots (2^{2^{n-1}}+1)=2^{2^n}-1.\]

\item 记费马数$F_k=2^{2^k}+1,k\geq 0$.证明:当$m>n$时,有$F_n\mid (F_m-2)$.
\end{enumerate}
\end{example}
\begin{proof}
\begin{enumerate}
\item 左边乘以$(2-1)$并多次利用平方差公式.

\item 由$F_m-2=F_{m-1}\cdot F_{m-2}\cdots F_1\cdot F_0$,由$m>n$可知$F_n$必出现在等式右边.
\end{enumerate}
\end{proof}

\begin{example}
设$n$为大于1的正整数.证明:存在从小到大排列后成等差数列(即从第二项起,每一项与它前面那项的差为常数的数列)的$n$个正整数,它们中任意两项互素.
\end{example}
\begin{proof}
考虑下面的$n$个数
\[n!+1,2\times (n!)+1,\cdots,n\times (n!)+1.\]
它们组成公差为$n!$的等差数列.

记$p\mid i\times (n!)+1$和$j\times (n!)+1\, (1\leq i<j\leq n)$,则
\[p\mid (j\times (n!)+1)-(i\times (n!)+1)=(j-i)\times n!,\]
则$p\mid n!$,于是$p\mid 1$,则$p=1$.
\end{proof}


\begin{example}
设$a$、$b$、$c$、$d$都是整数,且$a\neq c,a-c\mid ab+cd$.证明: $a-c\mid ad+bc$.
\end{example}
\begin{proof}
利用
\[(ad+bc)-(ab+cd)=d(a-c)-b(a-c)=(d-b)(a-c),\]
由$a-c\mid ab+cd$可知$a-c\mid ad+bc$.
\end{proof}

 

\begin{example}

\end{example}
\begin{proof}

\end{proof}

\begin{example}
(上海1989年高二数学竞赛)设$a,b,c$为满足不等式$1<a<b<c$的整数,且$(ab-1)(bc-1)(ca-1)$能被$abc$整除,求所有可能数组$(a,b,c)$.
\end{example}
\begin{proof}
因为
\[
\left( ab-1 \right) \left( bc-1 \right) \left( ca-1 \right) =a^2b^2c^2-abc\left( a+b+c \right) +ab+bc+ca-1,
\]
因为$abc\mid (ab-1)(bc-1)(ca-1)$,所以存在正整数$k$,使得
\[ab+bc+ca-1=kabc.\]
于是\[
k=\frac{1}{a}+\frac{1}{b}+\frac{1}{c}-\frac{1}{abc}<\frac{1}{a}+\frac{1}{b}+\frac{1}{c}<\frac{3}{a}<\frac{3}{2},
\]
则$k=1$.

若$a\geq 3$,则$
1=\frac{1}{a}+\frac{1}{b}+\frac{1}{c}-\frac{1}{abc}<\frac{1}{3}+\frac{1}{4}+\frac{1}{5}=\frac{47}{60}
$,矛盾.

又$a>1$,则$a=2$,此时有$2b+2c-1=bc$,即
\[
1=\frac{2}{b}+\frac{2}{c}-\frac{1}{bc}<\frac{2}{b}+\frac{2}{c}<\frac{2}{b}+\frac{2}{b}=\frac{4}{b},
\]
所以$0<b<4$,于是$b=3,c=5$.
\end{proof}

\begin{example}
(1992年第33届IMO)求所有的整数$a,b,c$,满足条件

(i) $1<a<b<c$;

(ii) $(a-1)(b-1)(c-1)$整除$abc-1$.
\end{example}
\begin{proof}
由题意可知$a\geq2,b\geq3,c\geq4$,故
\[
\frac{a-1}{a}=1-\frac{1}{a}\ge \frac{1}{2},\quad\frac{b-1}{b}=1-\frac{1}{b}\ge \frac{2}{3},\quad\frac{c-1}{c}=1-\frac{1}{c}\ge \frac{3}{4},
\]
所以$abc\le 4\left( a-1 \right) \left( b-1 \right) \left( c-1 \right)$,于是
\[
S=\frac{abc-1}{\left( a-1 \right) \left( b-1 \right) \left( c-1 \right)}<4.
\]

下面进行分类讨论:当$S=1$时,有$abc-1=\left( a-1 \right) \left( b-1 \right) \left( c-1 \right)$,即$a+b+c=ab+bc+ca$.但由$a<ab,b<bc,c<ca$可知$a+b+c<ab+bc+ca$,矛盾.

当$S=2$时,有$abc>abc-1=2\left( a-1 \right) \left( b-1 \right) \left( c-1 \right)$,则$a,b,c$均为奇数,再由$1<a<b<c$得$a\geq3,b\geq5,c\geq7$.

若$b\geq7$,则$c\geq9$,从而
\[
\frac{\left( a-1 \right) \left( b-1 \right) \left( c-1 \right)}{abc}\ge \frac{2}{3}\cdot \frac{6}{7}\cdot \frac{8}{9}=\frac{32}{63}>\frac{1}{2},
\]
矛盾,所以只能有$a=3,b=5$,则$16(c-1)=15c-1$,解得$c=15$.

当$S=3$时,有$abc>abc-1=3\left( a-1 \right) \left( b-1 \right) \left( c-1 \right)$.

若$a\geq3$,则$b\geq4,c\geq5$,则
\[
\frac{\left( a-1 \right) \left( b-1 \right) \left( c-1 \right)}{abc}\ge \frac{2}{3}\cdot \frac{3}{4}\cdot \frac{4}{5}=\frac{2}{5}>\frac{1}{3},
\]
矛盾,所以只能有$a=2$.

若$b\geq 5$,则
\[
\frac{\left( a-1 \right) \left( b-1 \right) \left( c-1 \right)}{abc}\ge \frac{1}{2}\cdot \frac{4}{5}\cdot \frac{5}{6}=\frac{1}{3},
\]
矛盾,故只能有$b=3,4$.

若$b=3$,有$6(c-1)=6c-1$,无解.若$b=4$,有$9(c-1)=8c-1$,解得$c=8$.

综上所述,解只能为$(a,b,c)=(3,5,15)$或$(2,4,8)$.
\end{proof}


\begin{definition}{}{dy2}
设$a_1,a_2,\cdots,a_n$是$n(n\geq 2)$个整数,若整数$d$是它们之中每个数的因数,则称$d$是$a_1,a_2,\cdots,a_n$的一个\textbf{公因数}.
\end{definition}
整数$a_1,a_2,\cdots,a_n$的公因数中最大的一个叫作\textbf{最大公因数},记作$(a_1,a_2,\cdots,a_n)$.若$(a_1,a_2,\cdots,a_n)=1$,则称$a_1,a_2,\cdots,a_n$\textbf{互质}或\textbf{互素},若$a_1,a_2,\cdots,a_n$中每两个整数互质,则称它们\textbf{两两互质}.

\begin{example}
$12,16$和$28$具有公约数$2$,最大公约数为$4$; $2,4$和$7$互质,$2,15$和$7$两两互质.
\end{example}

求最大公约数有多种方法,常见的有短除法、质因数分解法、辗转相除法、更相减损法.

\begin{theorem}{}{sl3}
若$a,b,c$是任意三个不全为$0$的整数,且
\[a=qb+c,\]
其中整数$q\neq 0$,则$a,b$与$b,c$具有相同的公因数,因而$(a,b)=(b,c)$.
\end{theorem}


\begin{theorem}{辗转相除法(Euclid's Algorithm)}{sl4}
设$a,b$是任意两个正整数,由带余数除法,我们有下面一系列等式:
\[\begin{aligned}
a&=q_1b+r_1,& &0<r_1<b,\\
b&=q_2r_1+r_2,& &0<r_2<r_1,\\
&\cdots\cdots\cdots\cdots\cdots &&\\
r_{n-2}&=q_nr_{n-1}+r_n,& &0<r_n<r_{n-1},\\
r_{n-1}&=q_{n+1}r_n+r_{n+1},& &r_{n+1}=0,
\end{aligned}\]
则$(a,b)=r_n$为最后一个不等于零的余数.
\end{theorem}
利用此算法不仅可以求出两个正整数的最大公因数,还是求解一次不定方程的基本工具.

\begin{example}
设$x,m,n$都是正整数且$x>1$.证明:
\[(x^m-1,x^n-1)=x^{(m,n)}-1.\]
\end{example}
\begin{proof}
由带余数除法可知
\begin{align*}
(x^m-1,x^n-1)&=(x^{q_1n+r_1}-1,x^n-1)\\
&=((x^{q_1n}-1)x^{r_1}-1,x^n-1)=(x^{r_1}-1,x^n-1),
\end{align*}
这里利用了$x-1$整除$x^k-1$, $k$为正整数.如此类推可知
\begin{align*}
(x^m-1,x^n-1)&=(x^n-1,x^{r_1}-1)=(x^{r_1}-1,x^{r_2}-1)\\
&=\cdots=(x^{r_{n-1}}-1,x^{r_n}-1)=(x^{r_nq_{n+1}}-1,x^{r_n}-1)\\
&=x^{r_n}-1=x^{(m,n)}-1.
\end{align*}
\end{proof}


\begin{example}
$n$是不小于$40$的偶数,试证明: $n$总可以表示成两个奇合数的和.
\end{example}
\begin{proof}
因为$n$是不小于$40$的偶数,所以, $n$的个位数字必为$0$、
$2$、$4$、$6$、$8$,现在对$n$的个位数字进行分类:
\begin{enumerate}
\item 若$n$的个位数字为$0$,则$n=15+5k$ ($k\geq 5$为奇数)

\item 若$n$的个位数字为$2$,则$n=27+5k$ ($k\geq 3$为奇数).

\item 若$n$的个位数字为$4$,则$n=9+5k$ ($k\geq 7$为奇数).

\item 若$n$的个位数字为$6$,则$n=21+5k$ ($k\geq 5$为奇数).

\item 若$n$的个位数字为$8$,则$n=33+5k$ ($k\geq 3$为奇数).
\end{enumerate}
综上所述,不小于$40$的任一偶数,都可以表示成两个奇合数之和.
\end{proof}

\begin{example}
求满足下列条件的最小的正整数$n$:对任意正整数$k>n$,存正整数$a,b,c$,使得$1<a<b<c,a+b+c=k$,且$(a,b)=(b,c)=(c,a)=1$.
\end{example}
\begin{solution}
由于$17=3+5+9$不满足题意(只能分解为三个奇数相加),取$n=17$,对于$k>17$,我们按照尽可能分解为两个连续的奇数的原则,分类讨论如下:
\begin{enumerate}
\item $k=4m+2=2+(2m-1)+(2m+1)$.

\item $k=4m+8=4+(2m+1)+(2m+3)$.

\item 若$k=6m+1$,则分为两类: $6(2s+1)+1=12s+7=3+(6s-1)+(6s+5)$;
$6\cdot 2s+1=12s=3+(6s-7)+(6s+5)$.

\item $6m+9=(2m+1)+(2m+3)+(2m+5)$.

\item 若$k=6m+5$,则分为两类: $6(2s+1)+5=12s+11=3+(6s+1)+(6s+7)$;
$6\cdot 2s+5=12s+5=3+(6s-5)+(6s+7)$.
\end{enumerate}
\end{solution}

值得一提的是,这与哥德巴赫猜想有关: 1742年,哥德巴赫给欧拉的信中提出了以下猜想:任一大于2的整数都可写成三个质数之和.但是哥德巴赫自己无法证明它,于是就写信请教赫赫有名的大数学家欧拉帮忙证明,他注意到:当$n$为偶数, $n=2+(n-2)$, 只需证明偶数$n-2$可以分解为两个质数的和;当$n$为奇数, $n=3+(n-3)$,只需证明偶数$n-3$可以分解为两个质数的和.因此欧拉在回信中提出另一等价版本,即任一大于2的偶数都可写成两个质数之和,由此开辟了当代民科的主流研究方向:哥德巴赫猜想.

\begin{note}
强哥德巴赫猜想:每个不小于$6$的偶数都是两个奇素数之和.

弱哥德巴赫猜想:每个不小于$9$的奇数都是三个奇素数之和.
\end{note}


维诺格拉多夫于1937年利用 Hardy-Littlewood 圆法以及自己所创的三角和估计方法证明了
\begin{theorem}{三素数定理}{sl}
每个充分大的奇数都是三个奇素数之和.
\end{theorem}
参考潘承洞《素数分布与哥德巴赫猜想》.

这里的强哥德巴赫猜想也被称为偶数哥德巴赫猜想,弱的则被称为奇数哥德巴赫猜想.最近, H. A. Helfgott 在2014年的文章中证明了三元哥德巴赫猜想(Ternary Goldbach Conjecture).他先基于圆法、大筛法、三角和估计等方法证明了对$n\geq 10^{27}$的一切奇数$n$,该猜想成立,同时又对 $n\leq 8.875\times 10^{30}$进行了计算机的验证,最终得到下面的结论:

\textbf{三元哥德巴赫定理.}每个不小于 $7$的奇数都是三个素数之和.

一个显然的推论是:

\textbf{推论.}每个大于$1$的整数都是至多四个素数之和.

至于强哥德巴赫猜想,目前最好的结果仍然是陈景润的 $(1+2)$.其无法攻克的难度即使在今天仍然存在,实际上, H. A. Helfgott 在证明了三元哥德巴赫定理之后说过: The strong conjecture remains out of reach.最后以潘承洞先生的原话来结束此猜想的讨论:

“二百多年来,虽然在研究哥德巴赫猜想中取得了这样重大的成就,要从$(1+2)$完全解决哥德巴赫猜想还有十分漫长的路程。或许,我们可以说,为了完全解决哥德巴赫猜想所需克服的困难可能比至今克服的更为巨大。因为依作者看来,不仅现有的方法不适用于来研究解决$(1+1)$,而且到目前为止还看不到可以沿着什么途径,利用什么方法来解决它。”

\begin{example}
设$p>2$为质数,若
\[
1+\frac{1}{2}+\frac{1}{3}+\cdots +\frac{1}{p-2}+\frac{1}{p-1}=\frac{a}{b},
\]
证明: $p\mid a$.
\end{example}
\begin{proof}
前后两项对应相加得
\begin{align*}
\frac{a}{b} &=1+\frac{1}{2}+\frac{1}{3}+\cdots +\frac{1}{\frac{p-1}{2}}+\frac{1}{\frac{p+1}{2}}+\cdots +\frac{1}{p-2}+\frac{1}{p-1}
\\
&=\frac{p}{1\cdot \left( p-1 \right)}+\frac{p}{2\cdot \left( p-2 \right)}+\cdots +\frac{p}{\frac{p-1}{2}\cdot \frac{p+1}{2}}=\frac{pM}{\left( p-1 \right) !}
\end{align*}
因为$p>2$为质数,所以$p$与$1$、$2$、$\cdots$、$p-1$都互质,从而与$(p-1)!$互质,因此约分后分子中仍有因数$p$,即$p\mid a$.
\end{proof}

 

\begin{theorem}{裴蜀定理(Bezout's Identity)}{sl12}
若$a,b$是任意两个不全为零的整数,则存在整数$x,y$,使得
\[xa+yb=(a,b).\]
即$a,b$的整数线性组合.一般地,若$a_1,a_2,\cdots,a_n$是任意$n$个不全为零的整数,则存在整数$x_1,x_2,\cdots,x_n$,使得
\[x_1a_1+x_2a_2+\cdots+x_na_n=(a_1,a_2,\cdots,a_n).\]
\end{theorem}

\begin{corollary}{}{sl12}
若$(a,c)=1,c\mid ab$,则$c\mid b$.
\end{corollary}
\begin{proof}
由$xa+yc=1$可知$xab+ybc=b$.
\end{proof}

\begin{definition}{}{dy3}
设$a_1,a_2,\cdots,a_n$是$n(n\geq 2)$个整数,若整数$d$是这$n$个数的倍数,则称$d$是这$n$个数的一个\textbf{公倍数}.
\end{definition}
这$n$个数的公倍数中最小的一个叫作\textbf{最小公倍数},记作$[a_1,a_2,\cdots,a_n]$.

\begin{theorem}{}{sl12}
$[a,b]=\frac{ab}{(a,b)}$.
\end{theorem}


\begin{example}
试证
\begin{enumerate}
\item 当$n>1$时, $1+\frac12+\cdots+\frac1n$都不是整数;

\item 当$n\geq 1$时, $\frac13+\frac15+\cdots+\frac1{2n+1}$也不是整数;

\item 对于任意正整数$m,n$,和$S(m,n)=\frac1m+\frac1{m+1}+\cdots+\frac1{m+n}$都不是整数.
\end{enumerate}
\end{example}
\begin{proof}
\begin{enumerate}
\item 设$2^k\leq n<2^{k+1}$,则$2^k$在$1,2,\cdots,n$的标准分解式中恰好出现一次.这是因为,若有$m\leq n,m\neq 2^k$,使得$2^k\mid m$,则$n\geq m=2^k\cdot l\geq 2^k\cdot 2>n$,矛盾!记$M$是不超过$n$的所有奇数的乘积,则
\[M\cdot 2^{k-1}\left(1+\frac12+\cdots+\frac1{n}\right)\]
不是整数,故$1+\frac12+\cdots+\frac1n$不是整数.

\item

\item

\end{enumerate}
\end{proof}


\begin{example}
设$n>0$,求$\binom{2n}{1},\binom{2n}{3},\cdots,\binom{2n}{2n-1}$的最大公因数.
\end{example}
\begin{solution}
设它们的最大公因数为$d$,因为
\begin{align*}
\binom{2n}{0}+\binom{2n}{1}+\binom{2n} {2}+\cdots+\binom{2n}{2n} &=2^{2n},\\
\binom{2n}{0}-\binom{2n}{1}+\binom{2n} {2}-\cdots+\binom{2n}{2n} &=0,
\end{align*}
两式相减得
\[\binom{2n}{1}+\binom{2n}{3}+\binom{2n} {5}+\cdots+\binom{2n}{2n-1} =2^{2n-1},\]
故$d\mid 2^{2n-1}$,可设$d=2^\lambda,\lambda\geq 0$.又设$2^k\mid n$,但$2^{k+1}\nmid n$,记为$2^k\parallel n$.下面我们证明$d=2^{k+1}$,由于
\[2^{k+1}\parallel \binom{2n}{1}=2n,\]
只需证明
\[2^{k+1}\mid \binom{2n}{j},\quad j=3,5,\cdots,2n-1.\]
设$n=2^kl$且$2\nmid l$,由
\[\binom{2n}{j}=\binom{2^{k+1}l}{j}=\frac{2^{k+1}l}{j}\cdot \binom{2^{k+1}l-1}{j-1},\quad j=3,5,\cdots,2n-1.\]

\[j\binom{2n}{j}=2^{k+1}l\cdot \binom{2^{k+1}l-1}{j-1},\quad j=3,5,\cdots,2n-1.\]
因为$j$是奇数,则$2\nmid j$,故$d=2^{k+1}$.
\end{solution}


\begin{example}
(2009年初联) $n$个正整数$a_1,a_2,\cdots,a_n$满足如下条件: $1=a_1<a_2<a_3<\cdots<a_n=2009$且$a_1,a_2,\cdots,a_n$中任意$n-1$个不同的数的算术平均数都是正整数,求$n$的最大值.
\end{example}
\begin{solution}
设$a_1,a_2,\cdots,a_n$中去掉$a_i$后剩下的$n-1$个数的算术平均数为正整数$b_i,i=1,2,\cdots,n$,即$b_i=\frac{(a_1+a_2+\cdots+a_n)-a_i}{n-1}$.

于是,对于任意的$1\leq i<i\leq n$,都有$b_i-b_j=\frac{a_j-a_i}{n-1}$,从而$n-1\mid (a_j-a_i)$.

由于$b_1-b_n=\frac{a_n-a_1} {n-1}=\frac{2008} {n-1}$是正整数,故$n-1\mid 2^3\times 251$.

由于
\begin{align*}
a_n-1 &=(a_n-a_{n-1})+(a_{n-1}-a_{n-2})+\cdots+(a_2-a_1)\\
&\geq (n-1)+(n-1)+\cdots+(n-1)=(n-1)^2,
\end{align*}
所以$(n-1)^2\leq 2008$,于是$n\leq 45$.结合$n-1\mid 2^3\times 251$,所以$n\leq 9$.

另一方面,令$a_1=8\times 0+1,a_2=8\times 1+1,a_3=8\times 2+1,\cdots,a_8=8\times 7+1,a_9=8\times 251+1$,则这$9$个数满足题设要求.

综上所述, $n$的最大值为$9$.
\end{solution}

\begin{example}
(2009年南昌八年级竞赛)已知$n$是大于$1$的整数.求证: $n^3$可以写成两个正整数的平方差.
\end{example}
\begin{proof}
\begin{align*}
n^3 &=\left( \frac{n}{2} \right) ^2\cdot 4n=\left( \frac{n}{2} \right) ^2\left[ \left( n+1 \right) ^2-\left( n-1 \right) ^2 \right]
\\
&=\left[ \frac{n}{2}\left( n+1 \right) \right] ^2-\left[ \frac{n}{2}\left( n-1 \right) \right] ^2.
\end{align*}
因为$n(n+1),n(n-1)$不仅大于$1$,而且均能被$2$整除,所以$\frac{n}{2}\left( n+1 \right),\frac{n}{2}\left( n-1 \right)$均为正整数.
\end{proof}

\begin{example}
(2011年江西初联)设$k$为正整数.证明:
\begin{enumerate}
\item 若$k$是两个连续正整数的乘积,则$25k+6$也是两个连续正整数的乘积;

\item 若$25k+6$是两个连续正整数的乘积,则$k$也是两个连续正整数的乘积.
\end{enumerate}
\end{example}
\begin{proof}
\begin{enumerate}
\item 如果$k$是两个连续正整数的乘积,设$k=n(n+1)$,其中$n$为正整数,则$25k+6=25n(n+1)+6=25n^2+25n+6=(5n+2)(5n+3)$为两个连续正整数的乘积;

\item 如果$25k+6$是两个连续正整数的乘积,设$25k+6=m(m+1)$,其中$m$为正整数,则$100k+25=4m^2+4m+1=(2m+1)^2$.
于是, $2m+1$是$5$的倍数,且$\frac{2m+1}{5}$是奇数.设$\frac{2m+1}{5}=2n+1$,则
\[4k+1=\left(\frac{2m+1}{5}\right)^2=(2n+1)^2.\]
因此$4k=(2n+1)^2-1=2n(2n+2)$,即$k=n(n+1)$,它是两个连续正整数的乘积.
\end{enumerate}
\end{proof}

 

\section{奇偶性}

\begin{theorem}{}{sl12}
奇数$\pm$奇数$=$偶数; 奇数$\pm$偶数$=$奇数;偶数$\pm$偶数$=$偶数.

奇数$\times$奇数$=$奇数;奇数$\times$偶数$=$偶数;偶数$\times$偶数$=$偶数.
\end{theorem}


\begin{example}
(1995年初联)试证:每个大于$6$的自然数$n$, 都可以表示为两个大于$1$且互质的自然数之和.
\end{example}
\begin{solution}
\begin{enumerate}
\item 当$n$是奇数,则$n=2+(n-2)$且$n-2$是大于$1$的奇数, $n-2$与$2$互质;

\item 若$n$是偶数,
\begin{enumerate}
\item 若$\frac{n}{2}$是偶数,则$n=\left(\frac{n}{2}-1\right)+\left(\frac{n}{2}+1\right)$且$\frac{n} {2}-1$与$\frac{n} {2}+1$是两个大于$1$的连续奇数,可见它们互质;


\item 若$\frac{n}{2}$是奇数,则$n=\left(\frac{n}{2}-2\right)+\left(\frac{n}{2}+2\right)$,而$\frac{n} {2}-2$与$\frac{n} {2}+2$都是大于$1$的奇数,而它们的最大公约数等于$\frac{n} {2}-2$与$4$的最大公约数,故它们互质,由此命题证毕.
\end{enumerate}
\end{enumerate}
直观上可以这样看,当$n>6$时,在$2,3,\cdots,n-2$中,必有一个数$A$与$n$互质$(2\leq A\leq n-2)$,记$B=n-A\geq 2$,有$n=A+B$.

此时, $A$与$B$必互质,否则$A$与$B$有公约数$d>1$,则$d$也是$n$的约数,从而$A$与$n$有大于$1$的公约数,与$A$、$n$互质矛盾.
\end{solution}

\begin{example}%(2007年克罗地亚数学竞赛题)
是否存在两条直角边长均为整数的直角三角形,其斜边长为$\sqrt{2022}$.
\end{example}
\begin{solution}
假设存在这样的直角三角形.设两直角边长度分别为$x,y$,则
\[x^2+y^2=2022.\]
又$2022$为偶数,故$x,y$的奇偶性相同.

若$x,y$均为偶数,则必有$4\mid (x^2+y^2)$,但$4\nmid 2022$,矛盾.所以$x,y$均为奇数设$x=2k+1,y=2l+1$ ($k,l$为非负整数),则原方程化为
\[(2k+1)^2+(2l+1)^2=2022,\]
化简得
\[k(k+1)+l(l+1)=505.\]
对于任何整数, $n(n+1)$为偶数,故上式左边为偶数,而右边为奇数,矛盾.
因此,这样的三角形不存在.
\end{solution}

\begin{example}
设$a$、$b$、$c$、$d$都是整数,且$ac$、$bc+ad$、$bd$都是某个整数$u$的倍数.证明:数$bc$和$ad$也是$u$的倍数.
\end{example}
\begin{proof}
由恒等式
\[(bc+ad)^2+(bc-ad)^2=4abcd=4(ac)(bd),\]
由条件可知$u^2\mid (bc-ad)^2$,则$u\mid bc-ad$.

设$bc+ad=ux,bc-ad=uy$,则$x^2+y^2=4\frac{ac}{u}\cdot\frac{bd}{u}$,故$x^2+y^2$为偶数,进而$x+y$与$x-y$都是偶数,所以$bc=\frac{x+y}{2}u,ad=\frac{x-y}{2}u$,于是$bc$和$ad$也是$u$的倍数.
\end{proof}

\begin{example}
已知$a,b$为正整数,且$a,b$之积为偶数.求证:必然存在整数$c,d$,使得$a^2+b^2+c^2=d^2$.

若$ab$为奇数,结论是否成立?请举例说明.
\end{example}
\begin{proof}
若$a,b$两个数均为偶数,则$a^2,b^2,a^2+b^2$也为偶数,则
\[d^2-c^2=a^2+b^2=(d-c)(d+c),\]
令$d-c=2,d+c=\frac{a^2+b^2}{2}$,则$c=\frac{a^2+b^2}{4}-1,d=\frac{a^2+b^2}{4}+1$.

若$a,b$两个数中一个为奇数,一个为偶数,则$a^2+b^2$为奇数,则
\[d^2-c^2=a^2+b^2=(d-c)(d+c),\]
令$d-c=1,d+c=a^2+b^2$,则$c=\frac{a^2+b^2-1}{2},d=\frac{a^2+b^2+1}{2}$.
\end{proof}

注:一个数可表示为平方差的形式当且仅当它被$4$除余数不为$2$.

 

\section{素数}

\begin{definition}{}{dy4}
对于大于$1$的整数,如果它的正因数只有$1$和它本身,则称它为\textbf{质数}或\textbf{素数};否则称为\textbf{合数}.
\end{definition}
$2$是唯一的偶质数.

\begin{example}
正整数$n$、$a$、$b$、$c$、$d$满足等式$ab=cd$,求证: $k=a^n+
b^n+c^n+d^n$是合数.
\end{example}
\begin{proof}
由正整数的质因数分解的唯一性,可求得这样的正整数$p,q,r,s$,使
$a=pq,b=rs,c=pr,d=qs$,则
\begin{align*}
k&=(pq)^n+(rs)^n+(pr)^n+(qs)^n\\
&=(p^n+s^n)(q^n+r^n).
\end{align*}
于是$k$是合数.
\end{proof}


\begin{example}
设$n$为大于$1$的正整数.证明: $n^4+4^n$是合数.
\end{example}
\begin{proof}
当$n$为偶数时, $n^4+4^n$是大于$2$的偶数,从而它是合数.当$n$为奇数时,设$n=2k+1$,则
\[n^4+4^n=(2k+1)^4+4\times (2^k)^4.\]
利用
\begin{align*}
x^4+4y^4=(x^2+2y^2)^2-4x^2y^2=(x^2-2xy+2y^2)(x^2+2xy+2y^2)
\end{align*}
可知此时$n$仍为合数.
\end{proof}

\begin{theorem}{}{sl12}
素数有无限多个.
\end{theorem}
\begin{proof}
(欧几里得)假设素数只有有限个,按从小到大的顺序记为$p_1,p_2,\cdots,p_n$,考虑$N=p_1p_2\cdots p_n+1$,若$N$是素数,矛盾.若$N$不是素数,由$p_i\nmid N$可知,存在$p>p_i$使得$p\mid N$,这也产生矛盾.
\end{proof}


\begin{theorem}{}{}
对于任意正整数$n$,存在$n$个连续的合数.
\end{theorem}
\begin{proof}
考察$(n+1)!+2,(n+1)!+3,(n+1)!+4,\ldots,(n+1)!+(n+1)$即可.
\end{proof}

\begin{theorem}{容斥原理}{sl12}
设$A_1,A_2,\cdots,A_n$为有限集合,用$|A_i|$表示集合$A_i$中的元素个数,那么
\begin{align*}
|A_1\cup A_2\cup \cdots\cup A_n|=&\sum_{i=1}^{n}|A_i|-\sum_{1\leq i<j\leq n}|A_i\cap A_j|\\
&+\sum_{1\leq i<j<k\leq n}|A_i\cap A_j\cap A_k|-\cdots+(-1)^{n-1}|A_1\cap A_2\cap\cdots\cap A_n|.
\end{align*}
\end{theorem}
\begin{proof}

\end{proof}

\begin{theorem}{}{sl12}
几乎所有的自然数都不是素数.
\end{theorem}
\begin{proof}
设$p_1$和$p_2$是$2$个素数, $X$是一个很大的自然数.

在$2,3,4,\cdots,X$中我们有如下三列与$p_1$和$p_2$有关的合数:
\[
2p_1,3p_1,\cdots ,\left[ \frac{X}{p_1} \right] p_1,\quad 2p_2,3p_2,\cdots ,\left[ \frac{X}{p_2} \right] p_2
\]

\[p_1p_2,2p_1p_2,\cdots ,\left[ \frac{X}{p_1p_2} \right] p_1p_2.\]
这三列数的个数分别是$\left[ \frac{X}{p_1} \right]-1,\left[ \frac{X}{p_2} \right]-1,\left[ \frac{X}{p_1p_2} \right]$,因此在$2,3,4,\cdots,X$中的素数以及与$p_1$和$p_2$无关的合数的个数为
\[X-1-\left( \left[ \frac{X}{p_1} \right] -1+\left[ \frac{X}{p_2} \right] -1 \right) +\left[ \frac{X}{p_1p_2} \right] \le \left( 1-\frac{1}{p_1} \right) \left( 1-\frac{1}{p_2} \right) X+3.\]
记$\pi (X)$为不超过$X$的素数的个数,则
\[\pi(X)\leq \left( 1-\frac{1}{p_1} \right) \left( 1-\frac{1}{p_2} \right) X+3.\]

设$p_1,p_2,\cdots,p_n$为前$n$个素数,进一步由容斥原理可知
\begin{align*}
\pi \left( X \right) \le & X-1-\sum_{i=1}^n{\left( \left[ \frac{X}{p_i} \right] -1 \right)}+\sum_{1\le i<j\le n}{\left[ \frac{X}{p_ip_j} \right]}+\cdots +\left( -1 \right) ^n\left[ \frac{X}{p_1p_2\cdots p_n} \right]
\\
<& n+X\left( 1-\sum_{i=1}^n{\frac{1}{p_i}}+\sum_{1\le i<j\le n}{\frac{1}{p_ip_j}+\cdots +\left( -1 \right) ^n\frac{1}{p_1p_2\cdots p_n}} \right)
\\
&+\left( \sum_{i=1}^n{1}+\sum_{1\le i<j\le n}{1+\cdots +1} \right)
\\
<&n+X\prod_{i=1}^n{\left( 1-\frac{1}{p_i} \right)}+\left[ 1+C_{n}^{1}++C_{n}^{2}+\cdots ++C_{n}^{n} \right]
\\
=& X\prod_{i=1}^n{\left( 1-\frac{1}{p_i} \right)}+2^n+n<X\prod_{i=1}^n{\left( 1-\frac{1}{p_i} \right)}+2^{n+1}.
\end{align*}
取$n+1=\left[ \log _2\sqrt{X} \right]$,则
\[
0<\frac{\pi \left( X \right)}{X}<\prod_{i=1}^{\left[ \log _2\sqrt{X} \right] -1}{\left( 1-\frac{1}{p_i} \right)}+\frac{2^{\left[ \log _2\sqrt{X} \right]}}{X},
\]
注意到
\[
\prod_{i=1}^{\infty}{\left( 1-\frac{1}{p_i} \right) ^{-1}}>\prod_{i\le N}{\left( 1-\frac{1}{p_i} \right) ^{-1}}=\prod_{i\le N}{\left( \sum_{k=0}^{\infty}{\frac{1}{p_{i}^{k}}} \right)}>\sum_{n=1}^N{\frac{1}{n}},
\]
由$\lim_{N\rightarrow \infty} \sum_{n=1}^N{\frac{1}{n}}$可知$\prod_{i=1}^{\infty}{\left( 1-\frac{1}{p_i} \right)}$.
因此$\lim_{x\to\infty}\frac{\pi \left( X \right)}{X}=0$.
\end{proof}

\begin{theorem}{$p_n$的上界估计}{sl12}
设全体素数按从小到大顺序排成的数列为$p_1=2,p_2=3,p_3,p_4,\cdots$,则有
\[p_n\leq 2^{2^{n-1}},\quad n=1,2,\cdots,\]

\[\pi(x)> \log_2\log_2x,\quad x\geq 2.\]
\end{theorem}
\begin{proof}
利用$p_n\leq p_1p_2\cdots p_{n-1}+1,n>1$,假设对$n\leq k$时命题均成立,则有$p_{k+1}\leq 2^{2^{0}}\cdot 2^{2^1}\cdots2^{2^{k-1}}+1=2^{2^k-1}+1<2^{2^k}$.

対任一$x\geq 2$,存在唯一的正整数$n$,使得$2^{2^{n-1}}\leq x<2^{2^n}$,于是
\[\pi(x)\geq \pi\left(2^{2^{n-1}}\right)\geq n,\]
再利用$x<2^{2^n}$即可.
\end{proof}


从多项式到拉格朗日插值公式,行列式.

\begin{enumerate}
\item 设$(3x-1)^7=a_7x^7+a_6x^6+\cdots+a_1x+a_0$,试求$a_0+a_2+a_4+a_6$的值.

\item 解不等式$||x+3|-|x-1||>2$.
\item 求$x^4-2x^3+x^2+2x-1$除以$x^2+x+1$的商式和余式.

\item 求方程$|xy|-|2x|+|y|=4$的整数解.
\end{enumerate}

 

\begin{theorem}{算术基本定理(Fundamental Theorem of Arithmetic)}{sl12}
任一大于$1$的整数$a$能够唯一地分解成
\[a=p_1^{\alpha_1}p_2^{\alpha_2}\cdots p_k^{\alpha_k},\qquad \alpha_i>0,i=1,\cdots,k\]
称为标准分解式,这里$p_1<p_2<\cdots<p_k$.
\end{theorem}

\begin{corollary}{质因数分解法}{sl12}
设$a,b$是任意两个正整数,且
\begin{align*}
a=p_1^{\alpha_1}p_2^{\alpha_2}\cdots p_k^{\alpha_k},\qquad \alpha_i\geq 0,i=1,\cdots,k,\\
a=p_1^{\beta_1}p_2^{\beta_2}\cdots p_k^{\beta_k},\qquad \beta_i\geq 0,i=1,\cdots,k,
\end{align*}

\[(a,b)=p_1^{m_1}p_2^{m_2}\cdots p_k^{m_k},\quad [a,b]=p_1^{n_1}p_2^{n_2}\cdots p_k^{n_k},\]
其中$m_i=\min\{\alpha_i,\beta_i\},n_i=\max\{\alpha_i,\beta_i\}$.
\end{corollary}


\begin{corollary}{约数个数定理}{sl12}
设$d(a)=\sum_{d\mid a}1$表示大于$1$的整数$a$的所有正约数的个数, $a$具有上面的分解式,则
\[d(n)=\prod_{i=1}^{k}(1+\alpha_i).\]
\end{corollary}
注意到每个$p_i^{\alpha_i}$中可取$p_i^{0},p_i^{1},\cdots,p_i^{\alpha_i}$共$1+\alpha_i$个因数即可.

\begin{corollary}{约数和定理}{sl12}
设$\sigma(a)=\sum_{d\mid a}d$表示大于$1$的整数$a$的所有正约数的和, $a$具有上面的分解式,则
\[\sigma(n)=\prod_{i=1}^{k}\frac{p_i^{\alpha_i+1}-1}{p_i-1}.\]
\end{corollary}
\begin{proof}
注意到
\[\sigma(n)=\prod_{i=1}^{k}\left(1+p_i^{1},\cdots,p_i^{\alpha_i}\right).\]
\end{proof}

\begin{definition}{高斯取整函数}{dy4}
设$x$是实数, $[x]$表示不超过$x$的最大整数,称为$x$的\textbf{整数部分},满足\[[x]\leq x<[x]+1\]
或\[x-1< [x]\leq x.\]
\end{definition}
记$\{x\}=x-[x]$为$x$的\textbf{小数部分}.

\begin{theorem}{}{sl12}
\begin{enumerate}
\item 对任意整数$m$,有$[x+m]=[x]+m$;

\item $[x]+[y]\leq [x+y]\leq [x]+[y]+1$,其中等号有且仅有一个成立.

\item 正整数$1,2,\cdots,N$中被正整数$a$整除的数的个数是$[N/a]$.
\end{enumerate}
\end{theorem}

引入一个记号.设$k$是非负整数, $a^k\parallel b$表示$b$恰好被$a$的$k$次方整除,即$a^k\mid b,a^{k+1}\nmid b$.

\begin{theorem}{}{sl12}
设素数$p$和正整数$n$满足$p^\alpha\parallel n!$,则
\[\alpha=\alpha(p,n)=\sum_{i=1}^{\infty}\left[\frac{n}{p^i}\right],\]
即为$n!$的标准分解式中素因数$p$的指数.
\end{theorem}

\begin{proof}
设想把$2,\cdots,n$都分解成标准分解式,则由算术基本定理, $\alpha$就是这$n-1$个分解式中$p$的指数之和,设其中$p$的指数是$r$的有$n_r$个$(r\geq 1)$,则
\begin{align*}
h&=n_1+2n_2+3n_3+\cdots\\
&=n_1+n_2+n_3+\cdots\\
&+n_2+n_3+\cdots\\
&+n_3+\cdots\\
&+\cdots\\
&=N_1+N_2+N_3+\cdots,
\end{align*}
其中$N_r=n_r+n_{r+1}+\cdots$恰好是$1,2,\cdots,n$这$n$个数中能被$p^r$除尽的个数,但由$N_r=\left[\frac{n}{p^r}\right]$,则
\[\alpha=\left[\frac{n}{p}\right]+\left[\frac{n}{p^2}\right]+\left[\frac{n}{p^3}\right]+\cdots.\]
\end{proof}

\begin{corollary}{}{sl12}
\[n!=\prod_{p\leq n}p^{\sum_{i=1}^{\infty}\left[\frac{n}{p^i}\right]}.\]
\end{corollary}


\begin{example}
求$28!$的十进制表示中末尾有多少个零?
\end{example}
\begin{solution}
只需求$28!$的标准分解式中$5$的指数,$\alpha(5,28)=\left[\frac{28} {5}\right]+\left[\frac{28}{5^2}\right]=6$,所以末尾有$6$个零.
\end{solution}


\begin{theorem}{}{sl12}
设$n$是正整数, $p$是素数,且$n$的$p$进制表示为
\[n=a_0+a_1p+a_2p^2+\cdots+a_kp^k,\qquad 0\leq a_i<p,\]
那么在$n!$的标准分解式中,质因数$p$的指数是
\[\alpha=\frac{n-S_n}{p-1},\]
这里$S_n=a_0+a_1+\cdots+a_k$.
\end{theorem}
\begin{proof}
利用求和换序可知
\begin{align*}
\alpha&=\sum_{i=1}^{\infty}\left[\frac{n}{p^i}\right]
=\sum_{i=1}^{\infty}\sum_{j=i}^{\infty}a_jp^{j-i}=\sum_{j=1}^{k}\sum_{i=1}^{j}a_jp^{j-i}
=\sum_{j=1}^{k}\left(a_j\frac{p^j-1}{p-1}\right)\\
&=\frac{1}{p-1}\left(\sum_{j=0}^{k}a_jp^j-\sum_{j=0}^{k}a_j\right)
=\frac{n-S_n}{p-1}.
\end{align*}
\end{proof}

\subsection{Lifting The Exponent Lemma}

记$v_p(x)$为可整除$x$的素数$p$的最大指数,即若$v_p(x)=\alpha$,则$p^{\alpha}\mid x$,而$p^{\alpha+1}\nmid x$.也可记为$p^{\alpha}\parallel x$,则有
\[v_p(xy)=v_p(x)+v_p(y),\quad v_p(x+y)\geq \min\{v_p(x),v_p(y)\}.\]

\begin{example}
能整除$63$的$3$的最大次方为$3^2$,则$3^2\parallel 63$或$v_3(63)=2$.
\end{example}

\begin{lemma}{}{sl12}
设$x$和$y$为整数(不必是正的), $n$为正整数. $p$为任一给定的素数,满足$(n,p)=1,p\mid x-y$,且$p\mid x,p\mid y$,则
\[v_p(x^n-y^n)=v_p(x-y).\]
\end{lemma}

\begin{lemma}{}{sl12}
设$x$和$y$为整数(不必是正的), $n$为正奇数. $p$为任一给定的素数,满足$(n,p)=1,p\mid x+y$,且$p\mid x,p\mid y$,则
\[v_p(x^n+y^n)=v_p(x+y).\]
\end{lemma}

\begin{theorem}{First Form of Lifting The Exponent Lemma (LTE)}{sl12}
设$x$和$y$为整数(不必是正的), $n$为正整数. $p$为任一给定的奇素数,满足$p\mid x-y$,且$p\mid x,p\mid y$,则
\[v_p(x^n-y^n)=v_p(x-y)+v_p(n).\]
\end{theorem}

\begin{theorem}{Second Form of LTE}{sl12}
设$x$和$y$为整数(不必是正的), $n$为正奇整数. $p$为任一给定的奇素数,满足$p\mid x+y$,且$p\mid x,p\mid y$,则
\[v_p(x^n+y^n)=v_p(x+y)+v_p(n).\]
\end{theorem}

当$p=2$时,我们有
\begin{theorem}{LTE for the case $p=2$}{sl12}
设$x$和$y$为奇数, 满足$4\mid x-y$,则
\[v_2(x^n-y^n)=v_2(x-y)+v_2(n).\]
\end{theorem}

\begin{theorem}{}{sl12}
设$x$和$y$为奇数, $n$为正偶数,则
\[v_2(x^n-y^n)=v_2(x-y)+v_2(x+y)+v_2(n)-1.\]
\end{theorem}

\subsubsection{总结}

设$p$为素数且$x$和$y$是整数(不必是正的)且它们都不能被$p$整除,则
\begin{enumerate}
\item 对于正整数$n$,

\begin{enumerate}
\item 若$p\neq 2$且$p\mid x-y$,则
\[v_p(x^n-y^n)=v_p(x-y)+v_p(n).\]

\item 若$p= 2$且$4\mid x-y$,则
\[v_2(x^n-y^n)=v_2(x-y)+v_2(n).\]

\item 若$p= 2$, $n$是偶数且$2\mid x-y$,则
\[v_2(x^n-y^n)=v_2(x-y)+v_2(x+y)+v_2(n)-1.\]
\end{enumerate}

\item 对于正奇数$n$,若$p\mid x+y$,则
\[v_p(x^n+y^n)=v_p(x+y)+v_p(n).\]

\item 对于正数$n$,若$(p,n)=1,p\mid x-y$,则
\[v_p(x^n-y^n)=v_p(x-y).\]
若$n$是奇数, $(p,n)=1,p\mid x+y$,则
\[v_p(x^n+y^n)=v_p(x+y).\]
\end{enumerate}


\begin{example}
找出方程$x^{2009}+y^{2009}=7^z$的所有正整数解.
\end{example}
\begin{solution}
先分解$2009=7^2\cdot 41$,由于$x+y\mid x^{2009}+y^{2009}$且$x+y>1$,则$7\mid x+y$,由LTE可知
\[v_7(x^{2009}+y^{2009})=v_7(x+y)+v_7(2009)=v_7(x+y)+2,\]
则$x^{2009}+y^{2009}=49\cdot k\cdot (x+y)$,其中$7\nmid k$.由条件可知$x^{2009}+y^{2009}=7^z$,这说明$x^{2009}+y^{2009}$的素因数只能是$7$,故$k=1$.因此$x^{2009}+y^{2009}=49(x+y)$.但是,当$\max\{x,y\}>1$时,此方程左边大于右边,显然$(1,1)$不是解.因此所给方程没有正整数解.
\end{solution}

参考: Lifting The Exponent Lemma (LTE), Version 6 - Amir Hossein Parvardi, April 7, 2011

\section{平方数}

\begin{definition}{}{dy4}
如果一个数是一个整数的平方,则称这个数是完全平方数. 如: $1$、 $4$、 $9$、 $\cdots$等都是完全平方数.
\end{definition}

完全平方数有下列性质:
\begin{enumerate}
\item 任何完全平方数的个位数只能是$0, 1, 4, 5, 6, 9$中的一个.

\item 奇完全平方数的十位数一定是偶数.

\item 偶完全平方数是$4$的倍数,奇完全平方数除以$4$的余数为$1$.

\item 完全平方数与完全平方数的积仍是完全平方数,完全平方数与
非完全平方数的积是非完全平方数.
\end{enumerate}

\begin{example}
证明在三阶行列式展开式
\[\begin{vmatrix}
a_{11} & a_{12} & a_{13} \\
a_{21} & a_{22} & a_{23} \\
a_{31} & a_{32} & a_{33}
\end{vmatrix}=a_{11}a_{22}a_{33}+a_{12}a_{23}a_{31}
+a_{13}a_{21}a_{32}-a_{13}a_{22}a_{31}-a_{11}a_{23}a_{32}
-a_{12}a_{21}a_{33}.\]
中的六项不可能同时为正.
\end{example}
\begin{solution}
六项的乘积为$-N^2$为负数.
\end{solution}

\begin{example}
(2012年江苏复赛)设$n$为正整数,且$n^4+2n^3+5n^2+12n+5$为完全平方数,求$n$.
\end{example}
\begin{solution}
注意到
\[(n^2+n+2)^2<n^4+2n^3+5n^2+12n+5<(n^2+n+4)^2,\]
因此$n^4+2n^3+5n^2+12n+5=(n^2+n+3)^2$,解得$n=1$或$n=2$.
\end{solution}

\begin{example}
(2010年江苏复赛)求所有正整数$x,y$,使得$x^2+3y$与$y^2+3x$都是完全平方数.
\end{example}
\begin{solution}
若$x=y$,则$x^2+3x$是完全平方数.因为$x^2<x^2+3x<x^2+4x+4=(x+2)^2$,所以$x^2+3x=(x+1)^2$,所以$x=y=1$.

若$x>y$,则$x^2<x^2+3y<x^2+3x<x^2+4x+4= (x+2)^2$.因为$x^2+3y$是完全平方数,所以$x^2+3y=(x+1)^2$,得$3y=2x+1$,由此可知$y$是奇数,设$y=2k+1$,则$x=3k+1$, $k$是正整数.

又$y^2+3x=4k^2+4k+1+9k+3=4k^2+13k+4$是完全平方数,且
\[(2k+2)^2=4k^2+8k+4<4k^2+13k+4<4k^2+16k+16=(2k+4)^2,\]
所以$y^2+3x=4k^2+13k+4=(2k+3)^2$,得$k=5$,从而求得$x=16,y=11$.

若$x<y$,同$x>y$的情形可求得$x=11,y=16$.

综上所述, $(x,y)=(1,1),(11,16),(16,11)$.
\end{solution}

 

\begin{example}
(2015年北大自招)整数$x,y,z$满足$xy+yz+zx=1$,则$(1+x^2)(1+y^2)(1+z^2)$可能取到的值为
\begin{tasks}(4)
\task $16900$
\task $17900$
\task $18900$
\task 前三个答案都不对
\end{tasks}
\end{example}
\begin{solution}
注意到$1+x^2=xy+yz+zx+x^2=(x+y)(x+z)$,则$(1+x^2)(1+y^2)(1+z^2)=\left[ (x+y)(y+z)(z+x)\right]^2$必为完全平方数.令\[\begin{cases}x+y=2,\\y+z=5,\\z+x=13,\end{cases}\]解得\[\begin{cases}x=5,\\y=-3,\\z=8.\end{cases}\]经检验,这组解满足题意,此时$(1+x^2)(1+y^2)(1+z^2)=16900$.
\end{solution}


\begin{example}
(1986年初联)设$a$、$b$、$c$、$d$都是整数,且$m=a^2+b^2,n=c^2+d^2$, $mn$也可以表示成两个整数的平方和,求其形式.
\end{example}
\begin{solution}
$mn=(ac+bd)^2+(ad-bc)^2$.
\end{solution}


\begin{example}
(1997年初联)若正整数$x,y$满足$x^2+y^2=1997$,则$x+y$等于?
\end{example}
\begin{solution}
不妨设$x$为奇数, $y$为偶数,因为$x^2+y^2$的个位数字是$7$,因此$x^2,y^2$的个位数字必是$1,6$.所以$x$的个位数字为$1$或$9$, $y$的个位数字必是$4$或$6$.

又$1997\equiv 1\,(\bmod 4)$,则$x\equiv 1\,(\bmod 4),y\equiv 0\,(\bmod 4)$.由$x^2<1997$知$x<45$,因此$x$可能值为$1$、$9$、$21$、$29$、$41$.

经检验,仅当$x=29$时,有$y=34$,使$29^2+34^2=1997$,所以$x+y=29+34=63$.
\end{solution}

\begin{example}
(2014年全国高中数学联赛江苏赛区复赛一试)已知$a$、$b$、$c$、$d$均为整数,且$p=a^2+b^2$为素数,若$p\mid (c^2+d^2)$,证明: $\frac{c^2+d^2}{p}$可以表示为两个整数的平方和.
\end{example}
\begin{solution}
因为$p\mid c^2+d^2$,所以$c^2+d^2=pm$,其中$m$为整数.于是
\[m=\frac{c^2+d^2}{p}=\frac{(c^2+d^2)(a^2+b^2)}{p^2}
=\frac{(c+di)(c-di)(a+bi)(a-bi)}{p^2},\]
一方面,
\[m=\frac{(c+di)(c-di)(a+bi)(a-bi)}{p^2}
=\frac{(ca-db)^2+(da+cb)^2}{p^2},\]
另一方面,
\[m=\frac{(c+di)(c-di)(a+bi)(a-bi)}{p^2}
=\frac{(ca+db)^2+(da-cb)^2}{p^2},\]
注意到
\begin{align*}
(ca+db)(ca-db) &=c^2a^2-d^2b^2=(pm-d^2)a^2-d^2b^2\\
&=pma^2-d^2(a^2+b^2)=p(ma^2-d^2).
\end{align*}
因为$p$是素数,所以$ca+db$和$ca-db$中至少有一个数能被$p$整除.

当$ca-db$能被$p$整除时,令$ca-db=pt$, $t$是整数,根据(1),因为$m$是整数,所以$da+cb$也能被$p$整除.令$da+cb=ps$, $s$是整数,则$\frac{c^2+d^2}{p}=m=t^2+s^2$.

当$ca+db$能被$p$整除时,同理可证: $\frac{c^2+d^2}{p}$也可以表示为两个整数的平方和.
\end{solution}

%2017届全国高中数学联赛江苏复赛试题,最后一道函数题
2008年全国高中数学联赛江苏赛区复赛, 14, 15题
\begin{example}

\end{example}
\begin{solution}

\end{solution}


\begin{example}
(2019年北京中学生复赛初二邀请试题)试确定$2019$能否表示为两个整数的立方和.如果能,请写出一种表示;如果不能,请说明理由.
\end{example}
\begin{solution}
假设$A,B$为整数, $A^3+B^3=(A+B)(A^2+B^2-AB)=2019$, $A+B$是整数, $A^2+B^2-AB$是正整数.

将$2019$拆成两个正整数乘积,求解二元二次方程组:
\[2019=1\times 2019=2019\times 1=3\times 673=673\times 3=\cdots\]
\end{solution}

能用两种不同方法表示成两个立方数之和的最小正整数是
\[1729=1+12^3=9^3+10^3.\]
能用两种不同方法表示成两个四次方数之和的最小正整数是
\[635318657= 59^4+158^4=133^4+134^4.\]

\begin{example}
(2018年北京中学生复赛高一试题)
\begin{enumerate}
\item 证明: $2018$可表示为两个正整数的平方和;

\item 证明:存在这样的三角形,可把它分割为$2018$个全等的三角形.
\end{enumerate}
\end{example}
\begin{proof}
\begin{enumerate}
\item 若存在正整数$x$、$y$,使得$2018=x^2+y^2$,由于$2018$被$4$除余$2$,则$x$、$y$必同为奇数.

又奇数的平方的个位数必为$1$、$5$、$9$之一,故$x^2$、$y^2$的个位均为$9$,即$x$、$y$的个位数为$3$或$7$.

设$x\leq y$,则$1009\leq y^2<2018$.于是, $y$的可能值只有$33$、$37$、$43$.经计算得
\[2018=13^2+43^2.\tag{$\ast$}\]

\item 由式$(\ast)$构想:将直角边长为$13$、$43$的一个直角三角形引斜边上的高线,分该三角形为两个相似的直角三角形,其斜边分别为$13$、$43$,将斜边为$13$的直角三角形各边$13$等分,分成$169$个斜边为$1$的小直角三角形;将斜边为$43$的直角三角形各边$43$等分,分成$1849$个斜边为$1$的小直角三角形.合计共分割成了$169+1849=2018$个斜边为$1$的彼此全等的直角三角形.

因此,满足题设要求的三角形是存在的.
\end{enumerate}
\end{proof}


\begin{theorem}{费马平方和定理}{sl12}
质数$p$能表示为两个平方数之和的充分必要条件是$p=2$或$p=4k+1$.
\end{theorem}

\begin{theorem}{}{sl12}
设正整数$n=d^2m$, $m$无平方因数,则不定方程$n=x^2+y^2$有解的充分必要条件是$m$没有形如$4k+3$的素因数.
\end{theorem}

\begin{theorem}{拉格朗日四平方和定理}{sl12}
每一正整数都能表成四个整数的平方.
\end{theorem}

\begin{theorem}{1}{sl12}

\end{theorem}

%东南2005三平方和
\begin{example}
(2005年第二届东南赛)试求满足$a^2+b^2+c^2=2005$,且$a\leq b\leq c$的所有三元正整数数组$(a,b,c)$.
\end{example}
\begin{solution}

\end{solution}


\begin{example}
(2019年江西预赛)试求所有由互异正奇数构成的三元集$\{a,b,c\}$,使其满足: $a^2+b^2+c^2=2019$.
\end{example}
\begin{solution}

\end{solution}


\begin{example}

\end{example}
\begin{solution}

\end{solution}

\begin{example}
(1988年IMO)设非负整数$a,b$使得$\frac{a^2+b^2}{1+ab}$为整数.求证:这个整数必是某一整数的平方.
\end{example}
\begin{proof}
记$k=\frac{a^2+b^2} {1+ab}$,设其为正整数.如果$a=b$,可得$(2-k)a^2=k$,由此可知$k=1$,它当然是个平方数.

现不妨设$a>b\geq 0$.如果$b=0$,那么$k=a^2$,它是一平方数.因此可设$a>b>0$,现固定$b$,讨论下面的二次方程:
\[x^2-kbx+b^2-k=0.\]
已知它有一个根$a$,记另一个根为$a_1$,那么
\[a+a_1=kb,\qquad aa_1=b^2-k.\]
由前一式得$a_1=kb-a$,可见$a_1$是一整数.由第二式得出
\[a_1=\frac{b^2-k}{a}<\frac{b^2}{a}=\frac{b}{a}\cdot b<b.\]

我们指出$a_1\geq 0$.如若不然,由
\[0=a_1^2+b^2+(-a_1)bk-k\geq a_1^2+b^2>0,\]
得出矛盾,所以$a>b>a_1\geq 0$.由此得
\[k=\frac{a^2+b^2}{1+ab}=\frac{a_1^2+b^2}{1+a_1b}.\]

如果$a_1=0$,即得$k=b^2$是一平方数.现设$a_1>0$,这时$b>a_1>0$,对$k=\frac{b^2+a_1^2}{1+ba_1}$重复刚才的推理,可知存在整数$b_1$,满足$b>a_1>b_1\geq 0$,并使得
\[k=\frac{a_1^2+b_1^2}{1+a_1b_1}.\]
这样又回到了原来的情况,不过这时有$a>b>a_1>b_1$.
这个过程不可能无限地进行下去,必然有一个$a_i=0$或$b_j=0$,不论何者发生, $k$都是一个平方数.
\end{proof}

此证法是罗马尼亚选手给出的,并由此获得了本次竞赛特别奖.

\begin{example}

\end{example}
\begin{proof}

\end{proof}

\section{不定方程}


\begin{example}
求不定方程$3x+5y=31$的整数解.
\end{example}
\begin{solution}
由$x=\frac{31-5y}{3}=10-2y+\frac{y+1}{3}$,要使方程有整数解, $\frac{y+1}{3}$必须为整数.取$y=2$,得$x=7$,因此$x=7,y=2$是原方程的一组解,从而所有整数解为$x=7+5k,y=2-3k$,其中$k$为任意整数.
\end{solution}

\begin{theorem}{二元一次不定方程}{sl12}
不定方程$ax+by=c$有整数解的充要条件是$(a,b)\mid c$.设$x_0,y_0$是方程$ax+by=c$的一组整数解,则此方程的一切整数解可表示为
\[\begin{cases}
x=x_0+\frac{b}{(a,b)}t,\\
y=y_0-\frac{a}{(a,b)}t,
\end{cases}\qquad t\in\mathbb{Z}.\]
\end{theorem}

\begin{note}
这里的特解$(x_0,y_0)$可由辗转相除法得到.
\end{note}

\begin{example}
求$7x+4y=100$的一切整数解.
\end{example}
\begin{solution}
\[x=-4t-100,\qquad y=7t+200.\]
\end{solution}

\begin{example}
不定方程$x_1+x_2+\cdots+x_m=n\,(m,n\in \mathbb{N}_+)$的\textbf{非负整数解}$(x_1,x_2,\cdots,x_m)$的个数为$C_{n+m-1}^{m-1}$;而\textbf{正整数解}$(x_1,x_2,\cdots,x_m)$的个数为$C_{n-1}^{m-1}$.
\end{example}

\begin{note}
一排有$n+m$个圆圈,最后一个被选取,放隔板;接着在剩下的$n+m-1$个圆圈中选取$m-1$个位置放隔板.在$n-1$个圆圈中选取$m-1$个位置放隔板.
\end{note}

\begin{example}
设$n$是给定的正整数,求
\[\frac{1}{n}=\frac{1}{x}+\frac{1}{y},\qquad x\neq y\]
的正整数解$(x,y)$的个数.
\end{example}
\begin{solution}
显然$x>n,y>n$,记$x=n+r,y=n+s$且$r\neq s$,则
\[\frac{1}{n}=\frac{1}{n+r}+\frac{1}{n+s}=\frac{2n+r+s}{(n+r)(n+s)},\]
故$n^2+(r+s)n+rs=2n^2+(r+s)n$,则$n^2=rs$.由于$n^2$的不同因数$r$共有$d(n^2)$个,但需要排除$r=n$这种情况,因此正整数解$(x,y)$的个数是$d(n^2)-1$.
\end{solution}

\begin{example}
求不定方程$x+2y+3x=2012$的正整数解的组数.
\end{example}
\begin{solution}
设$(x,y,z)$为方程的正整数解,则$3z\leq 2009$,即$1\leq z\leq 669$,分别可得
\[x+2y=2009,2006,\cdots,5.\]
对应地, $y$的取值范围分别是
\[1\leq y\leq 1004,1\leq y\leq 1002,1\leq y\leq 1001,1\leq y\leq 999,\cdots,1\leq y\leq 2.\]
由于当$y$、$z$确定后, $x$的值唯一确定,所以正整数解共有
\begin{align*}
&(1004+1002)+(1001+999)+\cdots +(5+3)+2\\
&=2006+2000+\cdots+8+2\\
&=\frac{1}{2}(2006+2)\times 335=336340.
\end{align*}
综上可知,共有$336340$组正整数解.
\end{solution}

\subsection{构造法}

\begin{example}
证明不定方程$x^3+y^4=z^5$有无穷多个正整数解.
\end{example}
\begin{proof}
取$x=2^{8+20t},y=2^{6+15t},z=2^{5+12t}$,其中$t$为自然数.
\end{proof}

\begin{example}
证明:对任意整数$n$,方程
$x^2+y^2-z^2=n$有无穷多组整数解$(x,y,z)$.
\end{example}
\begin{solution}
由恒等式
\[(k+1)^2-k^2=2k+1,\qquad (k+1)^2-(k-1)^2=4k\]
可知,当$m$为奇数或$4$的倍数时,方程$a^2-b^2=m$有整数解$(a,b)$.

对于原方程,只需取$x$,使$x$与$n$的奇偶性相反(这样的$x$有无穷多个),从而利用上述命题,方程$y^2-z^2=n-x^2$有整数解,可知方程有无穷多组整数解.
\end{solution}

\begin{example}

\end{example}
\begin{solution}

\end{solution}


\subsection{母函数法(Generating function)}

\begin{example}
求不定方程$x_1+x_2+\cdots+x_m=n\,(m,n\in \mathbb{N}_+)$的非负整数解$(x_1,x_2,\cdots,x_m)$的个数.
\end{example}
\begin{solution}
考虑母函数
\[f(x)=(1+x+x^2+\cdots)^m=\frac{1}{(1-x)^m},\]
它的展开式中,每一个$x^n$均为$x^{x_1}x^{x_2}\cdots x^{x_n}=x^n$,且$x_1+x_2+\cdots+x_m=n\,(x_i\geq 0)$.
\end{solution}

 

\begin{example}
求不定方程$x+y+z=10,0\leq x\leq 3,0\leq y\leq 4,0\leq z\leq 5$的解的个数.
\end{example}
\begin{solution}
考虑母函数
\begin{align*}
f(x)&=(1+x+x^2+x^3)(1+x+x^2+x^3+x^4)(1+x+x^2+x^3+x^4+x^5)\\
&=\frac{1-x^4}{1-x}\frac{1-x^5}{1-x}\frac{1-x^6}{1-x},
\end{align*}
再展开求系数$f_{10}$.
\end{solution}

\begin{example}
求不定方程$x+2y+5z=n$的非负整数解$(x,y,z)$的个数.
\end{example}
\begin{solution}
考虑母函数
\begin{align*}
f(x)&=(1+x+x^2+\cdots)(1+x^2+x^4+\cdots)(1+x^5+x^{10}+\cdots)\\
&=\frac{1}{1-x}\frac{1}{1-x^2}\frac{1}{1-x^5},
\end{align*}
再展开求系数$f_{10}$.
\end{solution}

\begin{lemma}{}{}
\begin{align*}
(1+x)^n &=\sum_{k=0}^{n}C_n^kx^k, \\
\frac{1}{(1-x)^n} &=\sum_{k=0}^{\infty}C_{k+n-1}^{n-1}x^k.
\end{align*}
\end{lemma}
\begin{note}
利用$\frac{1}{1-x}=\sum_{k=0}^{\infty}x^k$,两边求$n-1$阶导即可.
\end{note}


\begin{example}
(2019年吉林预赛)求所有的正整数$n$,使得方程$\frac{1}{x_1^2}+\frac{1}{x_2^2}+\cdots+\frac{1}{x_n^2}=\frac{n+1}{x_{n+1}^2}$有正整数解.
\end{example}
\begin{solution}
\begin{enumerate}
\item 当$n=1$时,方程变为$\frac{1} {x_1^2}=\frac{2} {x_2^2}$,得$\frac{x_2}{x_1}=\sqrt{2}$,显然无正整数解.

\item 当$n=2$时,方程变为$\frac{1} {x_1^2}+\frac{1} {x_2^2}=\frac{3} {x_3^2}$,得$(x_2x_3)^2+(x_1x_3)^2=3(x_1x_2)^2$.

先证引理: $a^2+b^2=3c^2$无正整数解.

假设有一组正整数解$a,b,c$,不妨设$a,b,c$的最大公约数为$1$.由$a,b$为正整数,知$a^2\equiv 0\text{或}1\,(\bmod 3)$, $b^2\equiv 0\text{或}1\,(\bmod 3)$,又$3c^2\equiv 0\,(\bmod 3)$,故$a^2\equiv 0\,(\bmod 3)$, $b^2\equiv 0\,(\bmod 3)$,即$a\equiv 0\,(\bmod 3)$, $b\equiv 0\,(\bmod 3)$,从而$c\equiv 0\,(\bmod 3)$,这与"$a,b,c$的最大公约数为$1$"矛盾.引理得证.

由$a^2+b^2=3c^2$无正整数解可知此时原方程无整数解.

\item 当$n=3$时,方程变为$\frac{1} {x_1^2}+\frac{1} {x_2^2}+\frac{1} {x_3^2}=\frac{4} {x_4^2}$,由$3^2+4^2=5^2$,得$\frac{3^2}{(3\times 4\times 5)^2}+\frac{4^2}{(3\times 4\times 5)^2}=\frac{5^2}{(3\times 4\times 5)^2}$,即$\frac{1}{20^2}+\frac{1}{15^2}=\frac{1}{12^2}$,所以$\frac{1}{20^2\cdot 12^2}+\frac{1}{15^2\cdot 12^2}=\frac{1}{(12^2)^2}$,可得$\frac{1}{20^2}\left(\frac{1}{20^2}+\frac{1}{15^2}\right)
+\frac{1}{15^2\cdot 12^2}=\frac{1}{(12^2)^2}$,即
$\frac{1}{400^2}+\frac{1}{300^2}+\frac{1}{180^2}
=\frac{1}{144^2}$,故$\frac{1}{400^2}+\frac{1}{300^2}+\frac{1}{180^2}
=\frac{4}{288^2}$,这说明原方程有正整数解: $x_1=400,x_2=300,x_3=180,x_4=288$.

\item 当$n\geq 4$时, $\frac{1} {x_1^2}+\frac{1}{x_2^2}+\cdots+\frac{1}{x_n^2}=\frac{n+1}{x_{n+1}^2}$有正整数解: $x_1=400,x_2=300,x_3=180,x_4=x_5=\cdots=x_n=x_{n+1}=288$.
\end{enumerate}

综上所述, $n=1$或$n=2$时,原方程无正整数解; $n\geq 3$时,原方程有正整数解.即所求的$n$为$\{n\in \mathbb{N}^\ast|n\geq 3\}$.
\end{solution}

\begin{example}
任给$n\geq 2$,证明:存在$n$个互不相同的正整数,其中任意两个的和整除这$n$个数的积.
\end{example}
\begin{solution}
我们任取$n$个互不相同的正整数$a_1,\cdots,a_n$,选取参数$K=\prod_{1\leq i<j\leq n}(a_i+a_j)$,考虑$Ka_1,\cdots,Ka_n$即可.或者考虑$1(2n)!,2(2n)!,\cdots,n(2n)!$.
\end{solution}

\begin{example}

\end{example}
\begin{solution}

\end{solution}

\begin{example}

\end{example}
\begin{solution}

\end{solution}

\section{同余}

\begin{definition}{同余}{dy4}
如果$a-b$能被$m$整除,则称$a$与$b$对模$m$\textbf{同余},记为$a\equiv b (\bmod m)$.
\end{definition}

\begin{theorem}{}{sl12}
若$a_1\equiv b_1\,(\bmod m),a_2\equiv b_2\,(\bmod m)$,则
\begin{enumerate}
\item 加法. $a_1+a_2\equiv b_1+b_2 \,(\bmod m)$. (减法也成立)

\item 乘法. $a_1a_2\equiv b_1b_2 \,(\bmod m)$.

\item 若$ac\equiv bc \,(\bmod m),(m,c)=d$,则$a\equiv b \,(\bmod m/d)$.特别地,若$ac\equiv bc \,(\bmod m),(m,c)=1$,则$a\equiv b \,(\bmod m)$.

\item 幂.若$a\equiv b \,(\bmod m)$,则$a^2\equiv b^2 \,(\bmod m)$.进一步有$a^n\equiv b^n \,(\bmod m)$,其中$n$为正整数.
\end{enumerate}
\end{theorem}


\begin{example}
设整数$x$、$y$、$z$满足
\[(x-y)(y-z)(z-x)=x+y+z.\]
证明: $x+y+z$是$27$的倍数.
\end{example}
\begin{proof}
考虑$x$、$y$、$z$除以$3$所得的余数,如果$x$、$y$、$z$中任意两个对模$3$不同余,那么
\[x+y+z\equiv 0+1+2\equiv 0\,(\bmod 3)\]
但是$3\nmid (x-y)(y-z)(z-x)$,矛盾.

现在$x$、$y$、$z$中必有两个对模$3$同余,由对称性,不妨设$x\equiv y\,(\bmod 3)$这时$3\mid x+y+z$,于是
\[z\equiv -(x+y)\equiv -2x\equiv x\,(\bmod 3),\]
这表明$x\equiv y\equiv z\,(\bmod 3)$,从而$27\mid x+y+z$.
\end{proof}

\begin{example}
\begin{enumerate}
\item %(2007年克罗地亚数学奥林匹克题)
证明:对任意正整数$a$、$b$、$c$、$d$,整数$(a-b)(a-c)(a-d)(b-c)(b-d) (c-d)$被$12$整除. %%第三章16题, 2005年土耳其国家队选拔赛题

\item (思考题)证明:对于所有整数$a_1,a_2,\cdots,a_n,n>2$, $\prod_{1\leq i<j\leq n}{(a_j-a_i)}$能被$\prod_{1\leq i<j\leq n}{(j-i)}$整除.
\end{enumerate}
\end{example}
\begin{proof}

\[x=(a-b) (a-c)(a-d)(b-c)(b-d) (c-d),\]
只需证$3,4$均整除$x$.

由抽屉原理知, $a,b,c,d$四个数中至少有两个模$3$同余,这两个数之差必可被$3$整除,故$3\mid x$.

若$a,b,c,d$四个数中有两个模$4$同余,则$4\mid x$.

如若不然, $a,b,c,d$模$4$的余数均不相等,此时,这四个余数必为两奇两偶,其差的乘积可被$4$整除,故仍有$4\mid x$.

又因为$(3,4)=1$,故$12\mid x$.
\end{proof}


\begin{example}
证明当$n=9k\pm 4$时,不定方程$x^3+y^3+z^3=n$无解.
\end{example}
\begin{proof}
注意到$x^3\equiv 0,\pm 1\,(\bmod 9)$.
\end{proof}

\begin{example}
证明:不定方程
\[x^2+y^2-8z^3=6\]
没有整数解.
\end{example}
\begin{proof}
若$(x,y,z)$是方程的整数解,两边模$2$可知$x$、$y$同奇偶;再对两边模$4$可知$x$、$y$都为奇数,于是$x^2\equiv y^2\equiv 1\,(\bmod 8)$,这要求$6=x^2+y^2-8z^3\equiv 2\,(\bmod 8)$,矛盾.故方程没有整数解.
\end{proof}


\begin{theorem}{}{sl12}
设$m$为给定的正整数,则全部整数可分成$m$个集合,记作$K_0,K_1,\cdots,K_{m-1}$,其中$K_r(r=0,1,\cdots,m-1)$是由一切形如$qm+r\,(q=0,\pm1,\pm 2,\cdots)$的整数所组成的.这些集合具有下列性质:
\begin{enumerate}
\item 任一整数必包含在且仅在其中一个集合里面;
\item 两个整数同在一个集合当且仅当这两个整数
对模$m$同余.
\end{enumerate}
\end{theorem}

\begin{example}
对模$3$,有
\[\mathbb{Z}=\{\cdots,-5,-2,1,4,7,\cdots\}
=\{\cdots,-4,-1,2,5,8,\cdots\}=\{\cdots,-3,0,3,6,9,\cdots\}.\]
\end{example}

\begin{definition}{}{dy4}
上述定理中的$K_0,K_1,\cdots,K_{m-1}$叫做\textbf{模$m$的剩余类},一个剩余类中任一数叫做它同类的数的剩余.若$a_0,a_1,\cdots,a_{m-1}$是$m$个整数,并且其中任何两数都不同在一个剩余类里($m-1$个集合中各取一个数),则$a_0,a_1,\cdots,a_ {m-1}$叫做模$m$的一个\textbf{完全剩余系}.
\end{definition}


\begin{theorem}{}{sl12}
设$m$是正整数, $(a,m)=1$, $b$是任意整数,若$a_0,a_1,\cdots,a_ {m-1}$是模$m$的完全剩余系,则$aa_0+b,aa_1+b,\cdots,aa_ {m-1}+b$也是模$m$的完全剩余系.
\end{theorem}
\begin{proof}
只需证明$aa_0+b,aa_1+b,\cdots,aa_ {m-1}+b$两两不同余即可.

假设$aa_i+b\equiv aa_j+b\,(\bmod m)$,其中$i\neq j$.于是$aa_i\equiv aa_j\,(\bmod m)$,由$(a,m)=1$可知$a_i\equiv a_j\,(\bmod m)$,矛盾.
\end{proof}

\begin{definition}{欧拉函数(Euler's totient function)}{dy4}
欧拉函数 $\varphi(a)$是定义在正整数上的函数,它在正整数$a$上的值等于$0,1,2,\cdots,a-1$中与$a$互质的数的个数.
\end{definition}

比如$\varphi(1)=1$,而$\varphi(8)=4$,因为$1,3,5,7$均和$8$互质.


由裴蜀定理可知
\begin{corollary}{}{sl}
若$(a,m)=1$,存在$x$使得
\[ax\equiv 1\,(\bmod m),\]
则称$x$为$a$关于模$m$的逆,记作$a^{-1}$或$\frac{1}{a}\,(\bmod m)$,它们形成模$m$的一个同余类.
\end{corollary}
%\begin{proof}

%\end{proof}

\begin{example}
在模$5$的剩余类中,有
\[1\cdot 1\equiv 2\cdot 3\equiv 4\cdot 4\equiv 1.\]
\end{example}

 

 

\begin{definition}{}{dy4}
如果一个模$m$的剩余类里面的数与$m$互质,就把它叫做一个与模$m$互质的剩余类.在与模$m$互质的全部剩余类中,从每一类各任取一数所作成的数的集合,叫做模$m$的一个简化剩余系,任一简化剩余系包含整数的个数为$\varphi(m)$.
\end{definition}

\begin{theorem}{}{sl12}
若$(a,m)=1$,若$r_1,\cdots,r_ {\varphi(m)}$是模$m$的简化剩余系,则$ar_1,ar_2,\cdots,ar_ {\varphi(m)}$也是模$m$的简化剩余系.
\end{theorem}
\begin{proof}
由于$(a,m)=1,(r_i,m)=1$,则$(ar_i,m)=1$.若$ar_i\equiv ar_j\,(\bmod m)$,则$r_i\equiv r_j\,(\bmod m)$,矛盾.
\end{proof}

\begin{theorem}{}{sl12}
若$m_1,m_2$互质,则$\varphi(m_1m_2)=\varphi(m_1)\cdot\varphi(m_2)$.
\end{theorem}

\begin{theorem}{}{sl12}
\begin{enumerate}
\item $\varphi\left(p^{\alpha}\right)=p^{\alpha}-p^{\alpha-1}$.

\item 若$a=p_1^{\alpha_1}p_2^{\alpha_2}\cdots p_k^{\alpha_k}$,则
\[\varphi(a)=a\left(1-\frac{1}{p_1}\right)\left(1-\frac{1}{p_2}\right)\cdots
\left(1-\frac{1}{p_k}\right).\]
\end{enumerate}
\end{theorem}
\begin{proof}
$1,2,\cdots,p^{\alpha}$中被$p$整除的数的个数为$\left[\frac{p^{\alpha}}{p}\right]=p^{\alpha-1}$,因此与$p^{\alpha} $互质的数的个数等于$\varphi\left(p^{\alpha}\right)=p^{\alpha}-p^{\alpha-1}$.
\end{proof}

\begin{definition}{}{dy4}

\end{definition}

\begin{theorem}{Euler定理}{sl12}
设$m$是大于$1$的整数, $(a,m)=1$,则
\[a^{\varphi(m)}\equiv 1\,(\bmod m).\]
\end{theorem}
\begin{proof}
设$r_1,r_2,\cdots,r_ {\varphi(m)}$是模$m$的简化剩余系,则$ar_1,ar_2,\cdots,ar_ {\varphi(m)}$也是模$m$的简化剩余系,故
\[ar_1\cdot ar_2\cdots ar_ {\varphi(m)}\equiv r_1r_2\cdots r_ {\varphi(m)}\,(\bmod m),\]

\[a^{\varphi(m)}(r_1r_2\cdots r_ {\varphi(m)})\equiv r_1r_2\cdots r_ {\varphi(m)}\,(\bmod m).\]
但$(r_i,m)=1,i=1,2,\cdots,\varphi(m)$.因此$(r_1r_2\cdots r_ {\varphi(m)},m)=1$,故$a^{\varphi(m)}\equiv 1\,(\bmod m)$.
\end{proof}

\begin{theorem}{费马小定理}{sl12}
设$p$是素数,则
\[a^p\equiv a\,(\bmod p).\]
\end{theorem}
\begin{proof}
若$(a,p)=1$,由Euler定理得
\[a^{\varphi(p)}=a^{p-1}\equiv 1\,(\bmod p),\]
因此$a^p\equiv a\,(\bmod p)$.

若$(a,p)\neq 1$,则$p\mid a$,则$a^p\equiv a\,(\bmod p)$.
\end{proof}

 

\begin{example}
证明: $2019^{2019}\equiv 4\,(\bmod 17)$
\end{example}
\begin{proof}
注意到$(17,2019)=1$,由费马小定理可知$2019^{16}\equiv 1\,(\bmod 17)$,则$2019^{2016}=2019^{16\times 126}\equiv 1\,(\bmod 17)$,又$2019\equiv 13\,(\bmod 17)$,则$2019^3\equiv 13^3\equiv (-4)^3=-64\equiv 4\,(\bmod 17)$,因此$2019^{2019}\equiv 4\,(\bmod 17)$.
\end{proof}

\begin{example}
求所有的素数对$(p,q)$,使得$pq\mid p^p+q^q+1$.
\end{example}
\begin{proof}
若素数$p=q$,则$p^2\mid 2p^p+1$,则$p=1$,矛盾.

由对称性,不妨设$p<q$.若$p=2$,则$q$能整除$q^q+5$,则$q=5$,此时$(p,q)=(2,5)$.

若$p>2$,则$p,q$均为奇素数,由题意可知$p^p+1\equiv 0\,(\bmod q)$,故$p^{2p}\equiv 1\,(\bmod q)$,由费马小定理,有$p^{q-1}\equiv 1\,(\bmod q)$,则$p^ {(2p,q-1)}\equiv 1\,(\bmod q)$.

注意到$2\mid (2p,q-1)$且$(2p,q-1)\mid 2p$,则只能是下面两种情形.

情形一.若$(2p,q-1)=2$,则$p^2\equiv 1\,(\bmod q)$,所以$q\mid p+1$或$q\mid p-1$,这与$p\leq q-2$矛盾.

情形二.若$(2p,q-1)=2p$,则$q\equiv 1\,(\bmod p)$,于是
\[0\equiv p^p+q^q+1\equiv 1^q+1=2\,(\bmod p),\]
则$p=2$,矛盾.

综上所述,满足条件的$(p,q)=(2,5)$或$(5,2)$.
\end{proof}

\begin{theorem}{威尔逊(Wilson)定理}{sl12}
设$p$是素数,则$(p-1)!\equiv -1\,(\bmod p)$.
\end{theorem}
\begin{proof}
当$p=2$时结论成立.

当$p\geq 3$时,对每个$a$, $1\leq a\leq p-1$,有唯一的$a^{-1},1\leq a^{-1}\leq p-1$,使得$aa^{-1}\equiv 1\,(\bmod p)$.而$a\equiv a^{-1}\,(\bmod p)$等价于$a^2\equiv 1\,(\bmod p)$,即$a\equiv a^{-1}\equiv \pm 1\,(\bmod p)$,所以$p-3$个数$\{2,3,\cdots,p-2\}$可配为$\frac{p-3}{2}$对,每对$\{a,a^{-1}\}$满足$aa^{-1}\equiv 1\,(\bmod p)$.因此
\begin{align*}
(p-1)!&=2\cdot 3\cdots (p-2)(p-1)\\
&\equiv 1\cdot (p-1)\equiv -1\,(\bmod p).
\end{align*}
\end{proof}


\begin{example}
证明:存在无穷多个正整数$n$,使得$n^2+1\nmid n!$.
\end{example}
\begin{proof}
\textbf{引理1.}对素数$p>2$, $p\equiv 1\,(\bmod 4)$等价于存在$x\,(1\leq x\leq p-1)$使得$x^2\equiv -1\,(\bmod p)$.

\textbf{证.}充分性.因为对$1\leq x\leq p-1,(p,x)=1$,所以$x^{p-1}=(x^2)^{\frac{p-1}{2}}\equiv 1\,(\bmod p),(x^2)^{\frac{p-1}{2}}\equiv {(-1)}^{\frac{p-1}{2}}\,(\bmod p)$,所以$\frac{p-1}{2}$为偶数,即$p\equiv 1\,(\bmod 4)$.

必要性.当$1\leq x\leq p-1$时, $x,2x,\cdots,(p-1)x$构成模$p$的既约剩余系,所以,存在$1\leq a\leq p-1$,使得$ax\equiv -1\,(\bmod p)$.若不存在$a\,(1\leq a\leq p-1,a= x)$使得$ax\equiv -1\,(\bmod p)$,则这样的$a,x$共配成$\frac{p-1}{2}$对,则有${(-1)}^{\frac{p-1}{2}}\equiv (p-1)!\equiv -1\,(\bmod p)$,即$\frac{p-1}{2}$为奇数,与$p=4k+1$矛盾!所以,必存在$x\,(1\leq x\leq p-1)$使得$x^2\equiv -1\,(\bmod p)$.

\textbf{引理2.}形如$4k+1\,(k\in \mathbb{Z})$的素数有无限多个.

\textbf{证.}假设形如$4k+1$的素数只有$n$个: $p_1,p_2,\cdots,p_n$,则$p_1,p_2,\cdots,p_n$都不是$a=4(p_1p_2\cdots p_n)^2+1$的素因数.

设$q$是$a$的一个素因数,则有$(2p_1p_2\cdots p_n)^2\equiv -1\,(\bmod q)$,因存在$1\leq x\leq q-1$使$2p_1p_2\cdots p_n\equiv x\,(\bmod q)$,即$x^2\equiv -1\,(\bmod q)$.所以,由引理1知$q=4k+1$,矛盾!

所以,形如$4k+1$的素数有无限多个.

回到原题:由引理1,2知,存在无穷多个素数$p$,使得存在$x\,(1\leq x\leq p-1)$使得$x^2\equiv -1\,(\bmod p)$.即$p\mid x^2+1$,因$p>x$,所以, $p\nmid x!, x^2+1\nmid x!$,因这样的素数$p$有无穷多个,所以,相应的$x$也无穷多个.
\end{proof}


\section{同余方程}

\begin{theorem}{一次同余方程}{sl12}
一次同余方程
\[ax\equiv b\,(\bmod m),\quad (a,m)=1,\]
有解的充分必要条件是$(a,m)\mid b$,此时解的个数(对模$m$而言)是$d=(a,m)$.
\end{theorem}
\begin{proof}
设$x_0$是方程的一组解(特解),则通解为
\[x=x_0+m_1t,\quad m_1=\frac{m}{d},t=0,\pm1,\pm2,\cdots\]
此时对模$m$来说,可以写成
\[x\equiv x_0+km_1\,(\bmod m),\quad k=0,1,\cdots,d-1.\]
而且它们两两不同余.
\end{proof}

\begin{theorem}{中国剩余定理(孙子定理)}{sl12}
设$m_1,m_2,\cdots,m_k$是$k$个两两互质的正整数, $m=m_1m_2\cdots m_k,m=m_iM_i,i=1,2,\cdots,k$,则同余方程组
\[x\equiv b_1\,(\bmod m_1),x\equiv b_2\,(\bmod m_2),\cdots,x\equiv b_k\,(\bmod m_k)\]
的解是
\[x\equiv M'_1M_1b_1+M'_2M_2b_2+\cdots+M'_kM_kb_k\,(\bmod m),\]
其中$M'_iM_i\equiv 1\,(\bmod m_i),i=1,2,\cdots,k$.
\end{theorem}
%\begin{proof}

%\end{proof}

\begin{example}
孙子算经里所提出的问题之一如下:

“今有物不知其数,三三数之剩二,五五数之剩三,七七数之剩二,问物几何?” “答曰二十三”.
\end{example}
\begin{solution}
设$x$是所求物数,由题意可知
\[x\equiv 2\,(\bmod 3),x\equiv 3\,(\bmod 5),\cdots,x\equiv 2\,(\bmod 7).\]
于是$m_1=3,m_2=5,m_3=7$,则$m=m_1m_2m_3=105$, $M_1=35,M_2=21,M_3=15$,而$M'_1=2,M'_2=1,M'_3=1$,于是最少个数
\[x\equiv M'_1M_1b_1+M'_2M_2b_2+M'_3M_3b_3=140+63+30=233\equiv 23\,(\bmod 105).\]
\end{solution}


\section{质数模的同余方程}

实际上,我们可以把求解高次同余方程的问题归结为质数模的高次同余方程
\[f(x)=a_nx^n+a_{n-1}x^{n-1}+\cdots+a_0\equiv 0\,(\bmod p),\]
这里$p$是质数,而$a_n$不能被$p$整除.

\begin{theorem}{}{sl12}
设$k\leq n$,而$x\equiv x_i\,(\bmod p)\,(i=1,2,\cdots,k)$是
\[f(x)=a_nx^n+a_{n-1}x^{n-1}+\cdots+a_0\equiv 0\,(\bmod p),\]
的$k$个不同解,则对任何整数$x$,有
\[f(x)\equiv (x-x_1)(x-x_2)\cdots (x-x_k)f_k(x)\,(\bmod p),\]
其中$f_k(x)$是首项系数为$a_n$的$n-k$次多项式.
\end{theorem}
\begin{proof}
由多项式带余数除法可知
\[f(x)=(x-x_1)f_1(x)+r,\]
其中$f_1(x)$是首项系数为$a_n$的$n-1$次多项式而$r$是一常数.由条件可知$f(x_1)\equiv 0\,(\bmod p)$,故$r\equiv 0\,(\bmod p)$.因此对任何整数$x$都有
\[f(x)\equiv (x-x_1)f_1(x)\,(\bmod p).\]
令$x=x_i\,(i=2,\cdots,k)$,得
\[0\equiv f(x_i)\equiv (x_i-x_1)f_1(x_i)\,(\bmod p).\]
但$x_i\,(i=2,\cdots,k)$与$x_1$对模$p$不同余,而$p$是质数,故
\[f_1(x_i)\equiv 0\,(\bmod p),\quad i=2,\cdots,k.\]
用这里的$f_1(x)$代替上面推导过程中的$f(x)$,根据数学归纳法可知命题成立.
\end{proof}

\begin{corollary}{}{sl12}
对任何整数$x$,有
\[x^{p-1}-1\equiv[(x-1)(x-2)\cdots(x-(p-1))]\,(\bmod p).\]
\end{corollary}
\begin{proof}
由费马小定理可知$x\equiv i\,(\bmod p)\,(i=1,2,\cdots,p-1)$是$x^{p-1}-1\equiv 0\,(\bmod p)$的$p-1$个不同解.
\end{proof}

令$x=p$便得威尔逊定理.

\begin{theorem}{拉格朗日定理}{sl12}
同余方程
\[f(x)=a_nx^n+a_{n-1}x^{n-1}+\cdots+a_0\equiv 0\,(\bmod p),\]
的解数不超过它的次数$n$.
\end{theorem}
\begin{proof}
利用反证法,设它的解数超过$n$个,即至少有$n+1$个解,设为
\[x\equiv x_i\,(\bmod p)\,(i=1,2,\cdots,n,n+1).\]
由上面的定理可知
\[f(x)\equiv a_n(x-x_1)(x-x_2)\cdots (x-x_n)\equiv 0\,(\bmod p).\]
由于$f(x_{n+1})\equiv 0\,(\bmod p)$,则
\[a_n(x_{n+1}-x_1)(x_{n+1}-x_2)\cdots (x_{n+1}-x_n)\equiv 0\,(\bmod p).\]
但$p$为质数, $p$不整除$a_n$,则存在$x_i$使得$x_{n+1}-x_i\equiv 0\,(\bmod p)$,矛盾.
\end{proof}

\begin{corollary}{}{sl}
设整系数多项式$f(x)=a_nx^n+a_{n-1}x^{n-1}+\cdots+a_0$, $p$是素数且$n<p$,如果同余方程
\[f(x)=a_nx^n+a_{n-1}x^{n-1}+\cdots+a_0\equiv 0\,(\bmod p),\]
至少有$n+1$个互不同余的解,则$f(x)$模$p$恒为零,即所有系数$a_i\,(i=0,1,\cdots,n)$均被$p$整除.
\end{corollary}
这是因为,若$f(x)$模$p$不恒为零,它模$p$的次数不超过$n$,故同余方程至多有$n$个不同的解,矛盾.

\begin{example}
(改编自2017年北京中学生复赛高一试题) 设
\[
1+\frac{1}{2}+\frac{1}{3}+\cdots +\frac{1}{p-2}+\frac{1}{p-1}=\frac{a}{b},
\]
其中$p>3$为素数, $a,b$为自然数,且$(a,b)=1$,证明: $p^2\mid a$.
\end{example}
\begin{proof}
考虑$p-2$次多项式
\[f(x)=(x-1)(x-2)\cdots (x-p+1)-x^{p-1}+1.\]
由费马小定理可知,同余方程$f(x)\equiv 0\,(\bmod p)$有$p-1$个不同的解$x\equiv 1,2,\cdots,p-1\,(\bmod p)$,故$f(x)$的系数都能被$p$整除.特别地,常数项$(p-1)!+1$是$p$的倍数.


\[(x-1)(x-2)\cdots (x-p+1)=x^{p-1}-s_1x^{p-2}+\cdots+s_{p-1},\tag{$\ast$}\]
注意到$s_{p-1}=(p-1)!,s_{p-2}=(p-1)!\left(1+\frac{1}{2}+\cdots +\frac{1}{p-1}\right)$.由上可知系数$s_1,\cdots,s_{p-2}$都是$p$的倍数.

在$(\ast)$中取$x=p$可知
\[p^{p-2}-s_1p^{p-3}+\cdots+s_{p-3}p-s_{p-2}=0.\]
若$p>3$,则由上式及$p\mid s_{p-3}$可知$p^2\mid s_{p-2}$,
于是$p^2\mid (p-1)!\frac{a}{b}=a\frac{(p-1)!}{b}$且$(a,b)=1$,因此$p^2\mid a$.
\end{proof}

由此可得
\begin{theorem}{Wolstenholme定理}{sl12}
对任一素数$p\geq 5$,有
\[\sum_{k=1}^{p-1}\frac{(p-1)!}{k}\equiv 0\,(\bmod p^2).\]
\end{theorem}


\begin{example}
设$p>3$是素数,设
\[1+\frac{1}{2}+\cdots+\frac{1}{p-1}+\frac{1}{p}=\frac{r}{ps},\quad (r,s)=1,\tag{$\ast$}\]
则$p^3\mid r-s$.
\end{example}
\begin{proof}

\[[(x-1)(x-2)\cdots(x-(p-1))]=x^{p-1}-s_1x^{p-2}+\cdots-s_{p-2}x+s_{p-1},\tag{$\ast\ast$}\]
由韦达定理可知
\[s_{p-1}=(p-1)!,\quad s_{p-2}=(p-1)!\left(1+\frac{1}{2}+\cdots+\frac{1}{p-1}\right).\]

\[x^{p-1}-s_1x^{p-2}+\cdots-s_{p-2}x+s_{p-1}\equiv x^{p-1}-1\,(\bmod p),\]
而由威尔逊定理可知$s_{p-1}+1\equiv 0\,(\bmod p)$,于是同余方程
\[-s_1x^{p-2}+\cdots-s_{p-2}x\equiv 0\,(\bmod p)\]
有$p$个解,故$p\mid (s_1,s_2,\cdots,s_{p-2})$.

在$(\ast\ast)$中令$x=p$,得
\[p^{p-2}-s_1p^{p-3}+\cdots+s_{p-3}p-s_{p-2}=0.\]
由于$p>3$,则$s_{p-2}\equiv 0\,(\bmod p^2)$,由$(\ast)$可知
\[s_{p-2}=\frac{(p-1)!(r-s)}{ps}.\]
因为$s\mid (p-1)!$且$p\nmid \frac{(p-1)!}{s}$,故由$s_{p-2}\equiv 0\,(\bmod p^2)$可知整数$\frac{r-s}{p}$被$p^2$整除,故$p^3\mid r-s$.
\end{proof}


\begin{definition}{}{dy4}
1
\end{definition}


\section{二次剩余}

\begin{definition}{}{dy4}
设$p$为奇质数且$(a,p)=1$,如果同余方程
\[x^2\equiv a\,(\bmod p),\qquad (a,p)=1\]
有解,则称$a$是\textbf{模$p$的二次剩余};若无解,则称$a$是\textbf{模$p$的二次非剩余}.
\end{definition}

\begin{theorem}{欧拉判别法}{sl12}
设$p$为奇质数且$(a,p)=1$, $a$是模$p$的二次剩余的充要条件是
\[a^{\frac{p-1}{2}}\equiv 1\,(\bmod p),\]
$a$是模$p$的二次非剩余的充要条件是
\[a^{\frac{p-1}{2}}\equiv -1\,(\bmod p),\]
且若$a$是模$p$的二次剩余,则
\[x^2\equiv a\,(\bmod p),\qquad (a,p)=1\]
恰好有两个解.
\end{theorem}

 

 

为了便于讨论,我们引进一个表示模$p$的二次剩余、二次非剩余的符号——Legendre符号.

\begin{definition}{}{dy4}
设素数$p>2$.定义整变量$a$的函数
\[\left(\frac{a}{p}\right)=\begin{cases}
1, & \mbox{当$a$是模$p$的二次剩余}; \\
-1, & \mbox{当$a$是模$p$的二次非剩余}; \\
0, & \mbox{当}\, p\mid a.
\end{cases}\]
我们把$\left(\frac{a}{p}\right)$称为是\textbf{模$p$的Legendre符号}.
\end{definition}

\begin{theorem}{}{sl12}
Legendre符号具有以下性质:
\begin{enumerate}
\item $\left(\frac{a}{p}\right)=\left(\frac{a+p}{p}\right)$;

\item $\left(\frac{a}{p}\right)\equiv a^{\frac{p-1}{2}}\,(\bmod p)$;

\item $\left(\frac{ab}{p}\right)=
\left(\frac{a}{p}\right)\left(\frac{b}{p}\right)$;

\item 当$p\nmid d$时, $\left(\frac{a^2}{p}\right)=1$;

\item $\left(\frac{1}{p}\right)=1,\left(\frac{-1}{p}\right)
=(-1)^{\frac{p-1}{2}}$.

\item $\left(\frac{2}{p}\right)
=(-1)^{\frac{p^2-1}{8}}$.
\end{enumerate}
\end{theorem}


\begin{theorem}{高斯二次互反律}{sl12}
设$p,q$均为奇素数且$p\neq q$,则
\[\left(\frac{p}{q}\right)\cdot \left(\frac{q}{p}\right)
=(-1)^{\frac{p-1}{2}\cdot \frac{q-1}{2}}.\]
\end{theorem}

\section{勾股数}

源于直角三角形三边关系的二次不定方程
\[x^2+y^2=z^2\]
称为勾股数组(Pythagorean triple),常见的勾股数组有$(3,4,5),(5,12,13),(6,8,10),(7,24,25),(8,15,17)$等.

为了求解该不定方程,由平方的特性,只需考虑正整数解即可.若$(x,y)=d>1$,两端可以约去$d^2$,故可以再假设$(x,y)=1$.

此时$x,y$必为一奇一偶,因为若它俩都是奇数,则$x^2=4m+1,y^2=4n+1$,此时$x^2+y^2=4(m+n)+2$,而$z^2=4N$或$4N+1$,矛盾.

不妨设$x$是偶数, $y$是奇数,则

\begin{theorem}{勾股数组}{sl12}
不定方程$x^2+y^2=z^2$满足条件:

(1) $(x,y)=1$; \qquad (2) $x$为偶数,


的一切正整数解可以表示成
\[x=2ab,\quad y=a^2-b^2,\quad z=a^2+b^2,\]
这里$a,b$是一奇一偶且互质的正整数.
\end{theorem}
这被称为是本原勾股数组(Primitive Pythagorean Triple).

\begin{corollary}{}{sl12}
单位圆周上的一切有理点可以表示成
\[
\left( \pm \frac{2ab}{a^2+b^2},\pm \frac{a^2-b^2}{a^2+b^2} \right) \quad \text{和}\quad \left( \pm \frac{a^2-b^2}{a^2+b^2},\pm \frac{2ab}{a^2+b^2} \right).
\]
\end{corollary}

\begin{example}
设$n$是一个正整数.证明:存在$n$个彼此不全等的勾股三角形(边长都为整数的直角三角形),它们的周长都相等.
\end{example}
\begin{proof}
我们取$n$组本原勾股数组$(x_k,y_k,z_k),k=1,2,\cdots,n$,这里$x_k<y_k<z_k$,且$(x_k,y_k)=1$,则这$n$个数组确定的$n$个三角形彼此不相似,分别记$S_k=x_k+y_k+z_k$,并设$S_1,S_2,\cdots,S_n$的最小公倍数为$S$.现在令
\[a_k=\frac{S}{S_k}\cdot x_k,\quad b_k=\frac{S}{S_k}\cdot y_k,\quad c_k=\frac{S}{S_k}\cdot z_k,\]
则$(a_k,b_k,c_k),k=1,2,\cdots,n$确定的$n$个直角三角形彼此不全等,并且它们的周长都等于$S$.
\end{proof}


大约在1637年左右,法国学者费玛在阅读丢番图(Diophatus)《算术》拉丁文译本时,曾在第11卷第8命题旁写道: “将一个立方数分成两个立方数之和,或一个四次幂分成两个四次幂之和,或者一般地将一个高于二次的幂分成两个同次幂之和,这是不可能的.关于此,我确信已发现了一种美妙的证法 ,可惜这里空白的地方太小,写不下.”
\begin{theorem}{费马大定理}{sl12}
当整数$n>2$时,不定方程$x^n+y^n=z^n$没有正整数解.
\end{theorem}


\begin{example}

\end{example}
\begin{proof}

\end{proof}

 


\begin{example}[2010年江西高考数学试题]
证明以下命题:
\begin{enumerate}
\item[(1)] 对任一正整数$a$,都存在正整数$b,c(b<c)$,使得$a^2,b^2,c^2$成等差数列;

\item[(2)] 存在无穷多个互不相似的三角形$\triangle_n$,其边长$a_n,b_n,c_n$为正整数,且$a_n^2,b_n^2,c_n^2$成等差数列.
\end{enumerate}
\end{example}
\begin{proof}
\begin{enumerate}
\item[(1)] 易知$1^2,5^2,7^2$成等差数列,故$a^2,(5a)^2,(7a)^2$也成等差数列,所以对任一正整数$a$,都存在正整数$b=5a,c=7a,(b<c)$,使得$a^2,b^2,c^2$成等差数列.

【评注】第一问本来是用于“送分”的,有点像“脑筋急转弯”,用此考一下学生的“灵气”,但实际上考生多数舍近求远,在考场上却偏不会往此处考虑,成功者竟低于百分之一,考完之后便悔之无及,正所谓“早知灯是火,饭熟已多时”;

至于第二问,编题者是借助“不相似的三角形”来掩盖第一问中的“成比例”思想.


\item[(2)] 若$a_n^2,b_n^2,c_n^2$成等差数列,则有$b_n^2-a_n^2=c_n^2-b_n^2$,即
\[(b_n-a_n)(b_n+a_n)=(c_n-b_n)(c_n+b_n),
\cdots\cdots\text{\ding{172}}\]
下面采用构造法,选取关于$n$的一个多项式,例如$4n\left(n^2-1\right)$,使得它可按两种方式分解因式,由于
\[4n\left(n^2-1\right)=(2n-2)\left(2n^2+2n\right)=(2n+2)\left(2n^2-2n\right),\]
因此令
\[\begin{cases}
a_n+b_n=2n^2-2n, \\
b_n-a_n=2n+2,
\end{cases}\qquad \begin{cases}
c_n+b_n=2n^2+2n, \\
c_n-b_n=2n-2,
\end{cases}\]
可得
\[\begin{cases}
a_n=n^2-2n-1, \\
b_n=n^2+1,\\
c_n=n^2+2n-1,
\end{cases}\quad (n\geq 4)\cdots\cdots\text{\ding{173}}\]
易验证$a_n,b_n,c_n$满足\ding{172},因此$a_n^2,b_n^2,c_n^2$成等差数列.

当$n\geq 4$时,有$a_n<b_n<c_n$且$a_n+b_n-c_n=n^2-4n+1>0$,因此以$a_n,b_n,c_n$为边可以构成三角形.

其次,任取正整数$m,n$ ($m,n\geq 4$,且$m\neq n$),假若三角形$\triangle_m$与$\triangle_n$相似,则有:
\[\frac{m^2-2m-1}{n^2-2n-1}=\frac{m^2+1}{n^2+1}=\frac{m^2+2m-1}{n^2+2n-1}.\]
据比例性质有:
\begin{align*}
\frac{m^2+1}{n^2+1} &=\frac{m^2+2m-1}{n^2+2n-1}=\frac{m^2+2m-1-\left( m^2+1 \right)}{n^2+2n-1-\left( n^2+1 \right)}=\frac{m-1}{n-1},\\
\frac{m^2+1}{n^2+1} &=\frac{m^2-2m-1}{n^2-2n-1}=\frac{m^2-2m-1-\left( m^2+1 \right)}{n^2-2n-1-\left( n^2+1 \right)}=\frac{m+1}{n+1},
\end{align*}
所以$\frac{m+1}{n+1}=\frac{m-1}{n-1}$,由此可得$m=n$,与假设$m\neq n$矛盾.

即任两个三角形$\triangle_m$与$\triangle_n$ ($m,n\geq 4,m\neq n$)互不相似,所以存在无穷多个互不相似的三角形$\triangle_n$,其边长$a_n,b_n,c_n$为正整数且$a_n^2,b_n^2,c_n^2$成等差数列.
\end{enumerate}
\end{proof}

我对此有个很好的证明,但是时间不够了.

\begin{example}
求满足下列条件的最大正整数$n$,使得对这样的$n$,有唯一的正整数$k$,满足
\[\frac{2021}{4041}<\frac{n}{n+k}<\frac{2020}{4039}.\]
\end{example}

\begin{example}
(2010中等数学)将小于$100$的九个互异正整数分别填入一个$3\times 3$的方格表中,使得表格的每行三数、每列三数都成为勾股数组,你的填法是:
\begin{table}[!htbp]
\centering
\begin{tabular}{|p{0.8cm}|p{0.8cm}|p{0.8cm}|}
\hline
% after \\: \hline or \cline{col1-col2} \cline{col3-col4} ...
\phantom{15} & \phantom{36} & \phantom{39} \\ \hline
\phantom{20} & \phantom{48} & \phantom{52} \\ \hline
\phantom{25} & \phantom{60} & \phantom{65} \\
\hline
\end{tabular}
%\caption{}\label{}
\end{table}
\end{example}

(本题也是一道考查灵感的问题,如果不能看出问题的结构,而是依靠“拼凑”去做,那么完成这一道小题将会耗去很多时间,甚至徒劳无功.)

本题答案是:
\begin{table}[!htbp]
\centering
\begin{tabular}{|c|c|c|}
\hline
% after \\: \hline or \cline{col1-col2} \cline{col3-col4} ...
{15} & {36} & {39} \\ \hline
{20} & {48} & {52} \\ \hline
{25} & {60} & {65} \\
\hline
\end{tabular}
%\caption{}\label{}

\begin{tabular}{|c|c|c|}
\hline
% after \\: \hline or \cline{col1-col2} \cline{col3-col4} ...
{24} & {32} & {40} \\ \hline
{45} & {60} & {75} \\ \hline
{51} & {68} & {85} \\
\hline
\end{tabular}
\end{table}
\begin{solution}
任取两个不同的勾股数组$a,b,c$与$xyz$则在表M中,每行、每列的三数也都是勾股数组;这是由于,若$a^2+b^2=c^2$,可得$(ka)^2+(kb)^2=(kc)^2$;
\begin{table}[!htbp]
\centering
$M$:\quad \begin{tabular}{|c|c|c|}
\hline
% after \\: \hline or \cline{col1-col2} \cline{col3-col4} ...
$ax$ & $ay$ & $az$ \\ \hline
$bx$ & $by$ & $bz$ \\ \hline
$cx$ & $cy$ & $cz$ \\
\hline
\end{tabular}
%\caption{}\label{}
\end{table}
例如取$3,4,5$与$5,12,13$得到一种填法;

若取$3,4,5$与$8,15,17$,得到另一种填法;

(因勾股数有无穷多组,故不同的填法也有无穷多,而所列出的两种情况是各元素均小于$100$的填法).
\end{solution}

\begin{definition}{}{}

\end{definition}

\section{素数分布与黎曼假设}

\begin{verse}
有人曾经问希尔伯特,如果500年后能重回人间,他最希望了解的事情是什么?希尔伯特回答说:“我想知道,黎曼猜想解决了没有。”

美国数学家蒙哥马利则表示,如果有魔鬼答应让数学家用自己的灵魂来换取一个数学命题的证明,多数数学家将会选择黎曼猜想。

\end{verse}

\begin{theorem}{算术基本定理的解析形式: Euler乘积公式}{sl12}
若$s>1$,则有
\[\zeta(s)=\sum_{n=1}^{\infty}\frac{1}{n^s}=
\prod_p{\frac{1}{1-p^{-s}}},
\]
这里乘积中的$p$取遍所有素数.
\end{theorem}
\begin{proof}
由于$p\geq 2$,故对$s>1$均有
\[
\frac{1}{1-p^{-s}}=1+p^{-s}+p^{-2s}+\cdots,
\]
取$p=2,3,\cdots,m$,并将这些级数乘在一起,所得到的一般项为$2^{-a_2s}3^{-a_3s}\cdots m^{-a_ms}=n^{-s}$,其中$n=2^{a_2}3^{a_3}\cdots m^{a_m},a_i\geq 0$,当且仅当$n$没有大于$m$的素因子时,这样的$n$就会在此乘积中只出现一次,从而有
\[\prod_{p\leq m}{\frac{1}{1-p^{-s}}}=\sum_{(m)}\frac{1}{n^s},\]
右边的求和取遍素因子不超过$m$的所有正整数.

这些数包括所有不超过$m$的数,则
\[0<\sum_{n=1}^{\infty}\frac{1}{n^s}-\sum_{(m)}\frac{1}{n^s}<\sum_{m+1}\frac{1}{n^s},\]
等式最右边的极限为$0$.因此
\[\sum_{n=1}^{\infty}\frac{1}{n^s}=\lim_{m\to\infty}\sum_{(m)}\frac{1}{n^s}
=\lim_{m\to\infty}\prod_{p\leq m}{\frac{1}{1-p^{-s}}}=\prod_p{\frac{1}{1-p^{-s}}}.\]

(Euler 本人的证明)注意到
\[
\frac{1}{2^s}\sum_{n=1}^{\infty}{\frac{1}{n^s}}=\frac{1}{2^s}+\frac{1}{4^s}+\frac{1}{6^s}+\cdots,
\]
于是
\[
\left( 1-\frac{1}{2^s} \right) \sum_{n=1}^{\infty}{\frac{1}{n^s}}=\frac{1}{1^s}+\frac{1}{3^s}+\frac{1}{5^s}+\cdots,
\]
这时所有含有因子$2$的$\frac{1}{n^s}$项都被消去了,类似地,用$1-\frac{1}{3^s}$乘以上式,则所有含有因子$3$的$\frac{1}{n^s}$项也被消去了,如此类推,用$1-\frac{1}{p^s}$ ($p$为任意素数)乘以上式,最后只剩下$\frac{1}{1^s}=1$,即
\[
\prod_p{\left( 1-\frac{1}{p^s} \right)}\cdot \sum_{n=1}^{\infty}{\frac{1}{n^s}}=\frac{1}{1^s}=1.
\]
\end{proof}

\begin{lemma}{调和级数发散}{}
\[\lim_{n\to\infty}\left(1+\frac{1}{2}+\frac{1}{3}+\cdots+\frac{1}{n}\right)=\infty.\]
\end{lemma}
\begin{proof}
(Oresme)当$n$足够大时,注意到
\begin{align*}
&1+\frac{1}{2}+\frac{1}{3}+\frac{1}{4}+\frac{1}{5}+\frac{1}{6}+\frac{1}{7}+\frac{1}{8}+\cdots\\
&>1+\frac{1}{2}+\left(\frac{1}{4}+\frac{1}{4}\right)+\left(\frac{1}{8}+\frac{1}{8}+\frac{1}{8}+\frac{1}{8}\right)+\cdots\\
&=1+\frac{1}{2}+\frac{2}{4}+\frac{4}{8}+\cdots\\
&=1+\frac{1}{2}+\frac{1}{2}+\frac{1}{2}+\cdots\to\infty
\end{align*}
\end{proof}

\begin{corollary}{Euler对素数无穷个的证明}{sl12}
素数有无穷多个.
\end{corollary}
\begin{proof}
由于
\[\sum_{n\leq x}\frac{1}{n}\leq \prod_{p\leq x}\frac{1}{1-p^{-1}}=\prod_{p\leq x}\frac{p}{p-1}=\prod_{k=1}^{\pi(x)}\frac{p_k}{p_k-1},\]
而$p_k\geq k+1$,则
\[\frac{p_k}{p_k-1}=1+\frac{1}{p_k-1}\leq 1+\frac{1}{k}=\frac{k+1}{k},\]
因此
\[\pi(x)+1=\prod_{k=1}^{\pi(x)}\frac{k+1}{k}\geq \sum_{n\leq x}\frac{1}{n}\to\infty.\]
\end{proof}

\begin{corollary}{}{sl12}
Riemann $\zeta$函数$\zeta(s)$在$\mathrm{Re}(s)>1$没有零点.
\end{corollary}
\begin{proof}
设$\mathrm{Re}(s)=a$,由Euler乘积公式可知
\begin{align*}
\left| \zeta \left( s \right) \right| &=\prod_p{\frac{1}{\left| 1-p^{-s} \right|}}\ge \prod_p{\frac{1}{1+p^{-a}}}=\exp \left\{ -\sum_p{\ln \left( 1+p^{-a} \right)} \right\}
\\
&>\exp \left\{ -\sum_p{p^{-a}} \right\} >0,
\end{align*}
这里利用了$\ln(1+x)<x,x>0$且$\sum_p{p^{-a}}$在$a=\mathrm{Re}(s)>1$时收敛.
\end{proof}

\begin{lemma}{Basel problem}{sl}
\[1+\frac{1}{2^2}+\frac{1}{3^2}+\cdots+\frac{1}{n^2}+\cdots=\frac{\pi^2}{6}.\]
\end{lemma}
\begin{note}
巴塞尔问题是一个著名的数论问题,这个问题首先由皮耶特罗·门戈利在1644年提出,由莱昂哈德·欧拉在1735年解决.由于这个问题难倒了以前许多的数学家,欧拉一解出这个问题马上就出名了,当时他二十八岁.欧拉把这个问题作了一番推广,他的想法后来被黎曼在1859年的论文《论小于给定大数的质数个数》(On the Number of Primes Less Than a Given Magnitude)中所采用,论文中定义了黎曼$\zeta$函数,并证明了它的一些基本的性质.

这个级数的和大约等于1.644934(OEIS中的数列A013661).巴塞尔问题是寻找这个数的准确值,并证明它是正确的.欧拉发现准确值是$\pi^2/6$,并在1735年公布.他的证明还不是十分严密,真正严密的证明在1741年给出.

欧拉最初推导的方法是聪明和新颖的.他把有限多项式的观察推广到无穷级数,并假设相同的性质对于无穷级数也是成立的.当然,欧拉的想法不是严密的,还需要进一步证明,但他计算了级数的部分和后发现,级数真的趋于$\pi^2/6$,不多不少.这给了他足够的自信心,把这个结果公诸于众.

欧拉的方法是从正弦函数的泰勒级数展开式开始
\[\sin x=x-\frac{x^3}{3!}+\frac{x^5}{5!}-\frac{x^7}{7!}+\cdots\]
两边除以$x$,得
\[\frac{\sin x}{x}=1-\frac{x^2}{3!}+\frac{x^4}{5!}-\frac{x^6}{7!}+\cdots\]
而$\frac{\sin x}x=0$的根出现在$x=n\pi$,其中$n=\pm 1,\pm 2,\pm 3,\ldots$我们假设可以把这个无穷级数表示为线性因子的乘积,就像把多项式因式分解一样:
\begin{align*}
\frac{\sin x}{x}&=\left( 1-\frac{x}{\pi} \right) \left( 1+\frac{x}{\pi} \right) \left( 1-\frac{x}{2\pi} \right) \left( 1+\frac{x}{2\pi} \right) \left( 1-\frac{x}{3\pi} \right) \left( 1+\frac{x}{3\pi} \right) \cdots \\
&=\left( 1-\frac{x^2}{\pi ^2} \right) \left( 1-\frac{x^2}{4\pi ^2} \right) \left( 1-\frac{x^2}{9\pi ^2} \right) \cdots
\end{align*}

如果把这个乘积展开,并把所有$x^2$的项收集在一起,我们可以看到, $\frac{\sin x}x$的二次项系数为:
\[-\left(\frac{1}{\pi^2}+\frac{1}{4\pi^2}+\frac{1}{9\pi^2}+\cdots \right)=-\frac{1}{\pi^2}\sum_{n=1}^{\infty}\frac{1}{n^2}.\]
但从$\frac{\sin x}x$原先的级数展开式中可以看出, $x^2$的系数是$-\frac{1}{3!}=-\frac{1}{6}$.这两个系数一定是相等的;因此
\[-\frac{1}{\pi^2}\sum_{n=1}^{\infty}\frac{1}{n^2}=-\frac{1}{6}.\]
等式两边乘以$-\pi^2$就可以得出所有平方数的倒数之和.
\end{note}

 

经过大量的数值计算 , Legendre及 Gauss都猜测
\begin{theorem}{素数定理}{sl}
\[\pi(x)\sim \frac{x}{\ln x}.\]
\end{theorem}
而在1896年法国数学家阿达玛(Jacques Hadamard)和比利时数学家普森(Charles Jean de la Vallée-Poussin)先后独立给出证明.在1948年, 塞尔伯格(Atle Selberg)和保罗·埃尔德什(Paul Erd\H{o}s)首次给出素数定理的初等证明.

\begin{theorem}{Dirichlet's Theorem on Arithmetic Progressions}{sl}
设$a,b$是两个互质的整数且$b>0$,则在整数等差数列$a,a+b,a+2b,\cdots,a+nb,\cdots$中有无穷多个素数,这里$n$为自然数.
\end{theorem}

\begin{theorem}{等差数列中的素数分布}{sl}
用$\pi_a(x)$表示等差数列$a,a+b,a+2b,\cdots,a+nb,\cdots$中不超过$x$的素数个数,则有
\[\pi_a(x)\sim\frac{\pi(x)}{\varphi(b)}
\sim\frac{1}{\varphi(b)}\cdot\frac{x}{\ln x},\quad x\to\infty.\]
\end{theorem}
这些素数看起来像是平均分布在模$b$的$\varphi(b)$个简化剩余类中.

\begin{theorem}{陶哲轩和Ben Green, 2005}{sl}
存在任意长的素数等差数列.
\end{theorem}

\begin{note}
类似“7、37、67、97、127、157”这样完全由素数组成的等差数列叫做素数等差数列.
\end{note}

记Gamma函数
\[\Gamma (s)=\int_{0}^{\infty}x^{s-1}e^{-x}dx,\qquad \mathrm{Re}(s)>0,\]
当$n$为正整数时,我们有$\Gamma (n)=n!$.

Euler在1749年,依据对发散级数大胆的计算找到了
\begin{align*}
&1+2+3+4+5+\cdots=-\frac{1}{12},\\
&1^2+2^2+3^2+4^2+5^2+\cdots=0,\\
&1^3+2^3+3^3+4^3+5^3+\cdots=\frac{1}{120}
\end{align*}
基于Euler等人的结果, Riemann进行解析延拓,将函数推广到复数域.


给出了函数方程
\begin{theorem}{黎曼$\zeta$函数的函数方程}{sl}
\[\zeta(1-s)=2(2\pi)^{-s}\Gamma(s)\cos\left(\frac{\pi s}{2}\right)\zeta(s).\]
\end{theorem}
的正确证明.
\begin{note}
在函数方程中取$s=2n+1,n=1,2,3,\cdots$,由$\cos\left(\frac{\pi s}{2}\right)=0$可得$\zeta(s)$的平凡零点$\zeta(-2n)=0$.而且,若$s$是$\zeta(s)$的非平凡零点,则$1-s$也是$\zeta(s)$的非平凡零点.
\end{note}

由$\zeta(s)$在区域$\mathrm{Re}(s)>1$上收敛可知
\begin{corollary}{}{sl12}
$\zeta(s)$的非平凡零点都在区域$0\leq \mathrm{Re}(s)\leq 1$中.
\end{corollary}
这个区域被称为临界带,而实部为$\frac{1}{2}$的直线称为临界线.


利用伯努利数
\begin{theorem}{伯努利数}{sl}
记$B_0=1,B_1=-\frac{1}{2}$.当$n\geq 2$时,有
\[B_n=\sum_{k=0}^{n}C_n^kB_k.\]
\end{theorem}
则有$B_2=\frac{1}{6},B_4=-\frac{1}{30},B_6=\frac{1}{42},B_3=B_5=B_7=\cdots=0$.由此可得
\begin{theorem}{}{sl}
如果$n\geq 0$,则
\[\zeta(-n)=\begin{cases}
-\frac{1}{2}, & n=0, \\
-\frac{B_{n+1}}{n+1}, & n\geq 1.
\end{cases}\]
特别地,当$n\geq 1$时,有$\zeta(-2n)=0$.
\end{theorem}
在$\zeta(s)$的函数方程中取$s=2k$,则有
\begin{theorem}{}{sl}
如果$k$为正整数,则
\[\zeta(2k)=(-1)^{k+1}\frac{(2\pi)^{2k}B_{2k}}{2(2k)!}.\]
\end{theorem}


接着黎曼考虑完备化的函数
\[\xi(s)=\pi^{-s/2}\Gamma\left(\frac{s}{2}\right)\zeta(s),\]
将其写成更具对称性的著名形式
\[\xi(s)=\xi(1-s).\]
进一步,发现了基于黎曼$\zeta$函数的无穷乘积形式
\[\zeta(s)=\exp\left\{\frac{\gamma+\ln\pi}{2}s-\ln2\right\}
\frac{1}{s-1}\prod_\rho\left(1-\frac{s}{\rho}\right)
\prod_{n=1}^{\infty}\left(1+\frac{s}{2n}\right)e^{-\frac{s}{2n}}\]
中全体素数与全体零(极)点之间的对偶性,其中$\gamma=0.577\cdots$为欧拉常数,而$\rho$取遍$\zeta(s)$的全部虚零点.

黎曼证明了对于不大于$x$的素数个数$\pi(x)$的黎曼显式公式(Riemann's Explicit Formula)
\begin{theorem}{黎曼显式公式, 1859年}{sl}
\[\sum_{n=1}^{\infty}\frac{1}{n}\pi\left(x^{\frac{1}{n}}\right)
=\mathrm{Li}(x)-\sum_\rho\mathrm{Li}(x^\rho)
+\int_{x}^{\infty}\frac{dt}{t(t^2-1)\ln t}-\ln 2,\]
其中$\mathrm{Li}(x)$为对数积分, $\rho$取遍$\zeta(s)$的满足$0<\mathrm{Re}(x)<1$的所有零点(它们等于$\zeta(s)$的虚零点,称其为非平凡零点或者本性零点,对$\rho$求和是将$\rho$与$1-\rho$放在一起编组的.
\end{theorem}

\begin{definition}{M\"{o}bius函数}{sl}
M\"{o}bius函数定义如下: $\mu(1)=1$.当$n>1$时, $n=p_1^{\alpha_1}p_2^{\alpha_2}\cdots p_k^{\alpha_k}$,则
\[\mu(n)=\begin{cases}
(-1)^k, & \mbox{当} \alpha_1=\alpha_2=\cdots=\alpha_k=1\mbox{时}, \\
0, & \mbox{其它情况}.
\end{cases}\]
\end{definition}


\begin{theorem}{M\"{o}bius第二反转公式(M\"{o}bius inversion formula)}{sl}
设$F$和$G$是定义在$[1,=\infty)$上的函数,对$x\geq 1$,以下条件等价:
\begin{enumerate}
\item $F(x)=\sum_{n\leq x}G\left(\frac{x}{n}\right)$;
\item $G(x)=\sum_{n\leq x}\mu(n)F\left(\frac{x}{n}\right)$.
\end{enumerate}
\end{theorem}
可得
\begin{align*}
\pi(x)&=\sum_{n=1}^{\infty}\frac{\mu(n)}{n}J\left(x^{\frac{1}{n}}\right)\\
&=J(x)-\frac{1}{2}J\left(x^{\frac{1}{2}}\right)
-\frac{1}{3}J\left(x^{\frac{1}{3}}\right)-\frac{1}{5}J\left(x^{\frac{1}{5}}\right)
+\frac{1}{6}J\left(x^{\frac{1}{6}}\right)+\cdots
\end{align*}

\[J(x)=\mathrm{Li}(x)-\sum_\rho\mathrm{Li}(x^\rho)
+\int_{x}^{\infty}\frac{dt}{t(t^2-1)\ln t}-\ln 2,\]
其中$\mu(n)$为M\"{o}bius函数,
\[\mathrm{Li}(x)=\int_{0}^{x}\frac{du}{\ln u}=\lim_{\varepsilon\to 0^+}\left(\int_{0}^{1-\varepsilon}\frac{du}{\ln u}+\int_{1+\varepsilon}^{x}\frac{du}{\ln u}\right)\sim \int_{2}^{x}\frac{du}{\ln u}\]
为对数积分.由此公式Riemann提出了漂亮的猜想
\[\boxed{\text{Riemann猜想: Riemann $\zeta$函数$\zeta(s)$的虚零点的实部全为$\frac{1}{2}$.}}\]
假设此猜想成立,即$\mathrm{Re}(\rho)=\frac{1}{2}$,则由黎曼显式公式可得
\[\pi(x)=\mathrm{Li}(x)-\frac{1}{2}\mathrm{Li}\left(x^{\frac{1}{2}}\right)
-\sum_\rho\mathrm{Li}(x^\rho)+\cdots\]
这表明Riemann猜想等价于
\[\pi(x)=\mathrm{Li}(x)+O(x^{\frac{1}{2}}\ln x),\]

1896年, J. Hadamard和C. J. de la Vall\'{e}e Poussin同时独立证明了在直线$\mathrm{Re}(s)=1$上$\zeta(s)\neq 0$,并由此推出了素数定理$\pi(x)\sim\frac{x}{\ln x}$.

\begin{theorem}{J. Hadamard和C. J. de la Vall\'{e}e Poussin, 1896年}{sl}
对任意实数$t$,必有
\[\zeta(1+it)\neq 0.\]
\end{theorem}

然而素数定理
\[\pi(x)\sim \mathrm{Li}(x)\sim \frac{x}{\ln x}\]
说的是$\mathrm{Re}(\rho)<1$,这是远比Riemann猜想要弱的结果.

1981年,古尔登与戴密克尔验证了前十万亿个$\zeta$函数的零点都在临界线上.虽然这些计算有利于验证黎曼猜想,但也揭示了不满足黎曼猜想的特例很可能存在.

因此个别数学家开始对黎曼猜想产生怀疑,比如曾在黎
曼猜想研究上作出过重大成就,后来却表示"假如我们能够坚定地相信这个猜想是错误的,日子会过得更舒适些"的李特尔伍德,他不相信黎曼猜想的理由是:一个长期不能解决的分析领域中的猜想通常会被发现是错误的,一个长期不能解决的代数领域中的猜想则通常会被发现是正确的.


\begin{center}
\textbf{参考文献}
\end{center}

\begin{enumerate}
\item 初等数论,第三版,闵嗣鹤,严士健, 2007年

\item 初等数论,冯志刚,数学奥林匹克命题人讲座,2015年

\item 奥林匹克数学中的数论问题,沈文选,张垚,冷岗松,唐立华,湖南师大出版社, 2009年

\item 哈代数论,第6版,人民邮电出版社, 2010年

\item 华罗庚文集:数论卷2 (数论导引), 2010年

\item 数论基础,潘承洞,高等教育出版社, 2012年

\item 初等数论,潘承洞,北京大学出版社, 2003年

\item 数论概论,希尔弗曼,机械工业出版社, 2008年

\item 初等数论100例,柯召, 孙琦,哈尔滨工业大学出版社, 2011年

\item 数学竞赛研究教程,单墫,江苏教育出版社, 2009年
\end{enumerate}

\section{整点}

\begin{definition}{整点}{sl}
整点的定义:在平面直角坐标系中,横、纵坐标均为整数的点叫做整点,整点也叫格点. (类似地可定义空间直角坐标系中的整点.)
\end{definition}

\begin{theorem}{皮克定理(Pick's Theorem)}{sl}
整点多边形的面积公式:顶点都在整点上的简单多边形(即不自交的多边形),其面积为$S$,多边形内的整点数为$N$,多边形边上的整点数为$L$,则\[S=N+\frac{L}{2}-1.\]
\end{theorem}


\textbf{八皇后问题},是一个古老而著名的问题,是回溯算法的典型案例.
\begin{example}
在$8\times 8$格的国际象棋上摆放八个皇后,使其不能互相攻击,即任意两个皇后都不能处于同一行、同一列或同一斜线上,问有多少种摆法?
\end{example}
高斯认为有76种方案. 1854年在柏林的象棋杂志上不同的作者发表了40种不同的解,后来有人用图论的方法解出92种结果.

\begin{example}
投掷一次骰子,出现点数为$1,2,\cdots,6$的概率均为$\frac{1}{6}$,连续投掷$10$次,出现点数和为$30$的概率有多少?
\end{example}
\begin{solution}
母函数
\begin{align*}
f(x)&=(x+x^2+x^3+x^4+x^5+x^6)^{10}
=x^{10}\left(\frac{1-x^6}{1-x}\right)^{10}\\
&=x^{10}\cdot \sum_{i=0}^{10}(-1)^iC_{10}^ix^{6i}\cdot \sum_{i=0}^{\infty}C_{10+i-1}^9x^{i},
\end{align*}
$x^{30}$的系数
\[f_{30}=C_{29}^9-C_{23}^9\cdot C_{10}^1+C_{17}^9\cdot C_{10}^2-C_{11}^9\cdot C_{10}^3=2930455,\]
故所求概率为$P=\frac{2930455}{6^{10}}\approx 0.0485$.
\end{solution}


\begin{example}
求满足下列条件的最大正整数$n$,使得对这样的$n$,有唯一的正整数$k$,满足
\[\frac{2021}{4041}<\frac{n}{n+k}<\frac{2020}{4039}.\]
\end{example}

(1987年AIME) What is the largest positive integer $n$ for which there is a unique integer $k$ such that $\frac{8}{15} < \frac{n}{n + k} < \frac{7}{13}$?

Multiplying out all of the denominators, we get:

\begin{align*}104(n+k) &< 195n< 105(n+k)\\ 0 &< 91n - 104k < n + k\end{align*}
Since $91n - 104k < n + k$, $k > \frac{6}{7}n$. Also, $0 < 91n - 104k$, so $k < \frac{7n}{8}$. Thus, $48n < 56k < 49n$. $k$ is unique if it is within a maximum range of $112$, so $n = 112$.
\begin{solution}

\end{solution}


设$A$为$m\times n$非零矩阵, $A'$表示$A$的转置, $b$为$m$元列向量.证明:若线性方程组$Ax=b$有解,则$b$与$A'y=0$解空间正交.反之如何?请说明理由.

若线性方程组$Ax=b$有解$x_0$,则$Ax_0=b$, $x'_0A'=b'$,对于线性方程$A'Y=0$任一解$y_0$,则$A'y_0=0$,则$0=x'_0(A'y_0)=x'_0A'y_0=b'y_0$,即$b'y_0=0$,
因此$b$与$A'Y=0$的解空间正交.

反之,若$b$与$A'Y=0$的解空间正交,则对于线性方程$A'Y=0$任一解$y_0$,则$A'y_0=0$,且$b'y_0=0$,则
$\begin{pmatrix}
A' \\
b'
\end{pmatrix}y_0=0$,因此方程$A'y=0$与$\begin{pmatrix}
A' \\
b'
\end{pmatrix}y=0$同解,故$r(A')=r\begin{pmatrix}
A' \\
b'
\end{pmatrix}$,转置后得$r(A,b)=r(A)$.因此方程$Ax=b$有解.

 

posted on 2019-10-17 17:02  Eufisky  阅读(1186)  评论(0编辑  收藏  举报

导航